Anda di halaman 1dari 51

SOAL UKG 2015

1.
A.
B.
C.
D.

Kerajaan Islam pertama di Indonesia


kerajaan Aceh
Kerjaan Demak
Kerjaan Majapahit
kerajaan Samudra Pasai ()

2. Alkisah, ada sebuah kerajaan bernama Kerajaan Linggar dipimpin oleh seorang
ratu, bernama Ratu Sima. Ratu Sima mempunyai tiga orang putra, semuanya laki-laki.
Putra pertama dan keduanya lahir kembar, bernama Pangeran Seta dan Pangeran Seto.
Keduanya putra kesayangan Ratu Sima. Sementara putra ketiganya lahir cacat.
Matanya buta. Ia bernama Pangeran Rasi. Sejak kecil, Pangeran Rasi dibesarkan oleh
Mbok Tumi, dayang kerajaan yang ditugaskan mengasuh Pangeran Rasi. Mereka hidup
bahagia, hingga Pangeran Rasi menjadi pemuda yang baik dan santun budi pekertinya.
Kemarau terus melanda Kerajaan Linggar. Seluruh rakyat sangat berharap turunnya
hujan. Kerajaan Linggar kekeringan. Ada yang mengatakan, Kerajaan Linggar terkena
hukuman karena keserakahan Pangeran Seta dan Pangeran Seto yang menebang pohon
di hutan Randualas, yaitu hutan yang tumbuh di samping Kerajaan Linggar. Seluruh
rakyat tak ada yang berani melaporkan keserakahan kedua pangeran kembar itu.
Semua orang takut karena keduanya putra kesayangan ratu. (Dikutip dari Cermat
Berbahasa, Penerbit Tiga Serangkai) Kata 'menebang' berasal dari kata .............
A. nebang B. tebang () C. metebang D. tebangan
3. Menurut bentuknya, magnet buatan diberi nama - nama sebagai berikut, kecuali .
...
A. magnet U B. magnet batang C. magnet tesla () D. magnet ladam
4. Bacalah laporan hasil pengamatan berikut!
Objek pengamatan: Kerbau
Tempat : Di kandang kerbau Pak Sanif, Desa Mekarwangi
Waktu : Pukul 14.0015.30 Tanggal 21 Januari 2006
Hasil-hasil Pengamatan Gerakan kerbau sangat lamban. Namun, hewan ini adalah
pekerja yang ulet. Ia sanggup bekerja seharian penuh membajak sawah. Kerbau
merupakan binatang kandang yang terbesar. Ia punya tanduk panjang dan kuku yang
kuat. Namun, ia tidak galak. Ia senang bersahabat kepada seorang bocah sekalipun.
Kerbau juga sangat setia kepada majikannya. Jarang sekali ia mengamuk ataupun
berontak. Kerbau biasa hidup tertib. Ia pun tidak senang mengembara jauh dari
kandangnya. Karena itu, para penggembalanya dapat asyik tidur-tiduran tanpa rasa
takut kehilangan. (Dikutip dari Intisari Bahasa Indonesia, Penerbit Pustaka Setia)
Topik laporan hasil pengamatan tersebut adalah .
A. makanan kesukaan kerbau
B. ukuran tubuh kerbau
C. tingkah laku kerbau ()
D. tugas-tugas kerbau
5. Salah satu sifat perubahan dalam inovasi yaitu upaya penyusunan kembali
berbagai komponen yang ada dalam siustem dengan maksud untuk
menyesuaikan dengan tuntutan dan kebutuhan, disebut . . . .
A. Addition
B. Restructuring ()
C. Alternation
D. Substitution
6. Alkisah, ada sebuah kerajaan bernama Kerajaan Linggar dipimpin oleh seorang
ratu, bernama Ratu Sima. Ratu Sima mempunyai tiga orang putra, semuanya
laki-laki. Putra pertama dan keduanya lahir kembar, bernama Pangeran Seta dan
Pangeran Seto. Keduanya putra kesayangan Ratu Sima. Sementara putra ketiganya
lahir cacat. Matanya buta. Ia bernama Pangeran Rasi. Sejak kecil, Pangeran Rasi
dibesarkan oleh Mbok Tumi, dayang kerajaan yang ditugaskan mengasuh Pangeran
Rasi. Mereka hidup bahagia, hingga Pangeran Rasi menjadi pemuda yang baik dan
santun budi pekertinya. Kemarau terus melanda Kerajaan Linggar. Seluruh rakyat

sangat berharap turunnya hujan. Kerajaan Linggar kekeringan. Ada yang


mengatakan, Kerajaan Linggar terkena hukuman karena keserakahan Pangeran Seta
dan Pangeran Seto yang menebang pohon di hutan Randualas, yaitu hutan yang
tumbuh di samping Kerajaan Linggar. Seluruh rakyat tak ada yang berani melaporkan
keserakahan kedua pangeran kembar itu. Semua orang takut karena keduanya putra
kesayangan ratu. (Dikutip dariCermat Berbahasa, Penerbit Tiga Serangkai) Amanat
yang dapat kita petik dari cerita tersebut adalah
A.
B.
C.
D.

Kita tidak boleh menyombongkan kelebihan yang kita miliki. ()


Kita tidak boleh memamerkan kekurangan kita.
Kesabaran dan ketabahan hati akan mendatangkan kebaikan.
Kita harus membanggakan diri sendiri supaya percaya diri.

7.
A.
B.
C.
D.

Tahun 1945 nagasaki di bom oleh sekutu pada tanggal


7 Maret 1945
16 Juni 1945
21 April 1945
9 Agustus 1945 ()

8. Salah satu contoh kesulitan belajar bahasa yang termasuk dalam kekurangan
kemampuan kognitif adalah .
A. menangkap makna secara penuh ()
B. mengingat kembali kata-kata
C. membandingkan informasi yang diterima
D. merumuskan alternatif pemecahan masalah
9.

Apa yang dimaksud dengan amendemen?

A. Perubahan undang-undang atau penambahan terhadapnya.


()
B. Pembatalan undang-undang.
C. Pemberlakuan kembali undang-undang.
D. Pembubaran anggota parlemen yang sedang menyusun undang-undang.
10. Inovasi merupakan suatu perubahan baru yang secara kualitatif berbeda dengan
yang ada sebelumnya, disengaja, untuk meningkatkan kemampuan dalam mencapai
tujuan. Pernyataan tersebut dikemukakan oleh . . . .
A. Santosa S. Hamidjaja
()
B. Mattew B. Milers
C.Sumantri
Mulyani
D.Rabbin
11. Sebuah kubus panjang rusuk 1 sisinya 7 dm, maka volume kubus tersebut adalah
...
A. 323 dm3
B. 343 dm3 ()
C. 333 dm3
D. 363 cm3
12. Himpunan bilangan cacah yang merupakan kelipatan 6 , antara 18 dan 60 adalah . .
..
A. { 24, 30, 36, 42, 48, 54 } ()
B. { 16, 24, 30, 36, 42, 48, 54, 60 }
C. { 24, 30, 36, 48, 54 }
D. { 24, 30, 42, 48, 54 }

13. Kreativitas merupakan salah satu karakteristik perkembangan intelektual siswa


SD, yang artinya kemampuan untuk .
A. memecahkan masalah-masalah dalam kehidupan sehari-hari yang sering dilakukan dan
menghasilkan kepuasan kepada dirinya sendiri dan orang lain
B. penalaran yang menggunakan logika-logika yang dapat diterima oleh semua orang dan
menghasilkan penyelesaian persoalan untuk mengambil keputusan
C. berfikir tentang sesuatu dengan suatu cara yang baru dan tidak biasa serta
menghasilkan penyelesaian yang unik terhadap berbagai persoalan ()
D. mengembangkan ide-ide secara cerdas dalam rangka penyelesaian masalah-masalah yang
dihadapi dalam kehidupan masa sekarang maupun masa yang akan datang
14. Bagian Tanah pada gambar dibawah ini yang banyak mengandung humus
adalah ....
A. bagian E
B. bagian F
C. bagian G
D. bagian H ()
15. Sekitar tahun 6000 masehi, di pulau sumatra terdapat kerajaan...
A. sriwijaya ()
B. majapahit
C. samudra pasai
D. Demak
16. Pembelajaran Kontekstual dianggap pembelajaran yang bersifat mendidik, hal
itu disebabkan karena karakteristik pembelajaran tersebut adalah .
A. Membangkitkan minat siswa.
B. Merangsang siswa lebih aktif belajar.
C. Memberi kesempatan siswa berkreasi
D. Membiasakan siswa belajar sesuai dengan lingkungan. ()
17. Pernyataan di bawah ini merupakan karakteristik perkembangan peserta didik
SD/MI ditinjau dari aspek fisik, kecuali ....
A. menunjukkan variasi yang besar pada tinggi dan berat badan
B. memiliki keterampilan fisik untuk memainkan permainan
C. penambahan-penambahan dalam kemampuan motorik halus
D. memiliki kemampuan dalam mengangkat beban yang berat ()
18. Dalam menjalankan tugasnya, presiden dibantu oleh wakil presiden dan para
menteri. Siapakah yang menentukan dan memilih para menteri?
A. Rakyat
B. MPR.
C. DPR
D. Presiden. ()

19. Perkembangan perilaku moral dan perkembangan konsep moral merupakan


fase-fase perkembangan moral yang harus dicapai seorang anak. Pada fase
perkembangan perilaku moral, seorang anak belajar melalui cara-cara berikut,
kecuali .
A. coba-ralat (trial and error)
B. pendidikan langsung,
C. identifikasi
D. observasi ()
20. Ketika dalam pembelajaran Bahasa Indonesia, ketika ada siswa yang sangat
antusias memerankan tokoh dongeng atau cerita rakyat yang disukai berarti guru telah
menyajikan bahan pelajaran dengan menarik, dengan demikian guru dapat
mengindentifikasi minat siswa terhadap.
A. penampilan B. pekerjaan

C. sekolah ()

D. status

21. Berikut ini yang termasuk cara perkembangbiakan vegetatif buatan pada
tumbuhan yaitu . . . .A. rhizome B.geragih C.stolon D. mengenten ()
22. Sumbu simetri lipat pada segilima beraturan adalah ....
A. 4
B. 8
C. 5 ()
D. 10
23. Pernyataan di bawah ini yang merupakan karakteristik perkembangan peserta
didik SD/MI ditinjau dari aspek sosial adalah....
A. mulai menyukai teman sebaya sesama jenis ()
B. berperan serta dalam permainan logika
C. menyukai teman sebaya lawan jenis
D. dapat bekerja dalam durasi waktu yang lama
24. Hasil dari 63 x 33 adalah ....
A. 2.079 ()
B. 3.888
C. 3.777
D. 3.666
2
25. Luas segitiga 60 cm , dan tingginya 24 cm. Alas segitiga adalah . . . .
A. 4 cm
B. 5 cm
C. 6 cm
D. 8 cm
26. Pada anak usia sekolah dasar sering disebut usia berkelompok. Pernyataan
tersebut menunjukkan karakteristik perkembangan anak dalam aspek .
A. sosial ()
B. moral
C. intelektual
D. emosional

27.

Kutub magnet yang senama apabila didekatkan akan . . . .


A. tarik - menarik

B. tolak - menolak ()
C. berhimpitan
D. tetap
28.

29.

Untuk membelajarkan kompetensi dasar: mendeskripsikan bendabenda di


sekitar dan fungsi anggota tubuh dengan kalimat sederhana, maka
kemampuan
awal/ prasyarat yang harus dimiliki siswa adalah kemampuan .
A. membuat karangan deskriptif
B. melengkapi kalimat belum selesai
C. menyusun kalimat sederhana ()
D. membaca kalimat sederhana
Perhatikan soal-soal di bawah ini. 75
68 27
13
_
+ 58 71
Jawaban soal di atas menunjukkan salah satu kesalahan umum yang sering
dilakukan oleh anak usia SD yang berkesulitan belajar Matematika dalam hal .
A. proses penghitungan
B. memahami simbol
C. nilai tempat ()
D. bahasa dan membaca
30. Candi di bawah adalah peninggalan dari kerajaan...

A. Budha ()

B. Kristen

C. Hindu

D. Islam

31. Bacalah teks berikut untuk menjawab soal dibawah ini. Bakat menulis Sony sudah
muncul sejak usia 8 tahun. Kemahirannya tersebut bisa jadi merupakan warisan dari
ayahnya. Ayahnya memang terkenal sebagai seorang penulis yang karya-karyanya
sangat digemari orang. Banyak orang mengatakan bahwa ayahnya adalah seorang
penulis yang hebat. Hal ini disebabkan tulisan-tulisan ayahnya dinilai mampu
membahagiakan hati orang-orang yang membacanya. Kini, meskipun masih kelas VI
SD, Sony sudah menerbitkan belasan novel yang dikarangnya sendiri. Selain sudah
menerbitkan buku, Sony juga selalu menjadi juara dalam setiap lomba yang
diikutinya. Pernah ia menjadi juara dalam lomba menulis cerita anak tingkat kota
madya dan lomba menulis puisi anak tingkat provinsi Jawa Barat. Jika dihitung,
jumlah piagam dan piala yang berderet di rumahnya sudah mencapai lebih dari tiga
puluh buah. Semua prestasi Sony tersebut tentu saja diawali oleh perjuangan dan
ketekunan. Dorongan semangat dari ayahnya memang sangat berpengaruh. Namun,
prestasi Sony takkan dapat diraih tanpa adanya kemauan yang kuat dalam dirinya
untuk tetap menulis. Kelak, ia berharap bisa menjadi seorang penulis terkenal seperti
ayahnya. Ia pun berharap tulisan-tulisannya bisa bermanfaat bagi semua orang,
khususnya anak-anak. Apa yang menjadi harapan Sony di masa depan?
A. menerbitkan banyak buku
B. menjadi juara di setiap lomba
C. menjadi seorang penulis terkenal ()
D. menulis puisi yang bagus-bagus
32. Kerajaan Tarumanegara berdiri pada tahun...
A. 1971
B. 680 SM
C. 450 M ()
D. 420 SM
33. Salah satu lembaga tinggi negara yang anggotanya dipilih oleh rakyat lewat
Pemilihan Umum adalah . . . .
A. Dewan Perwakilan Rakyat Daerah ()
B. Mahkamah Agung
C. Mahkamah Konstitusi
D. Badan Pemeriksa Keuangan
34. FPB dari bilangan , 120, 160 dan 200 adalah ....
A. 40 ()

B. 80

C. 60

35. Yang termasuk distribusi tidak langsung adalah ...


A. tukang nasi goreng
B. toko sepatu bata
C. pedagang ()
D. Kantin
36.

n adalah . . . .
A.
B.
C.
D.

13
23
27 ()
37

D. 90

37. Binatang menyusui = . . . .


A. aves
B. reptilia
C. mamalia ()
D. dekapoda
38. Kemampuan awal/prasyarat yang harus dimiliki siswa sebelum mempelajari
kompetensi dasar: Mendeskripsikan gejala (peristiwa) alam yang terjadi negara
tetangga, adalah .
A. mendeskripsikan gejala/peristiwa alam yang terjadi di Indonesia
B. mendeskripsikan gejala/peristiwa alam yang terjadi di Eropa
C. mendeskripsikan gejala/peristiwa alam yang terjadi di Asia ()
D. mendeskripsikan gejala/peristiwa alam yang terjadi di Afrika
39. Hak, wewenang, dan kewajiban daerah untuk mengatur pemerintahannya
sendiri sesuai dengan peraturan perundang-undangan yang berlaku disebut . . . .
A. pemerintahan otoriter
B. pemerintah daerah
C. otoritas daerah
D. otonomi daerah ()
40. Kompetensi yang harus dimiliki guru (pendidik) berdasarkan Permendiknas
Nomor 16 tahun 2007 adalah kompetensi ..
A. Kepribadian, pedagogik, professional, dan manajerial.
B. Kepribadian, pedagogik, professional, dan social. ()
C. Kepribadian, pedagogik, professional, dan kewirausahaan.
D. Pedagogik, professional, manajeria, dan sosial.
41. Standar nasional pendidikan yang berkaitan dengan perencanaan, pelaksanaan,
dan pengawasan kegiatan pendidikan pada tingkat satuan pendidikan,
kabupaten/kota, provinsi, atau nasional agar tercapai efisien dan efektivitas
penyelenggaraan pendidikan disebut .
A. Standar Proses.
B. Standar Kompetensi Lulusan.
C. Standar Sarana dan Prasarana.
D. Standar Pengelolaan. ()
42. Sejumlah kemampuan yang harus dimiliki peserta didik dalam mata pelajaran
tertentu sebagai rujukan untuk menyusun indikator kompetensi adalah :
A. Standar Kompetensi
B. Kompetensi Dasar ()
C. Kompetensi Dasar Mata Pelajaran
D. Standar Kompetensi Lulusan
43. Susi dan Indri berlatih menari di tempat yang sama. Susi berangkat setiap 3 hari sekali
dan Indri berangkat setiap 4 hari sekali. Pada tanggal 5 April keduanya berangkat
bersama-sama. Keduanya berangkat bersama-sama lagi pada tanggal
.

A. 16 April
B. 17 April ()
C. 18 April
D. 19 April
44. Dalam proses pembelajaraan Matematika, khususnya ketika guru mengaitkan materi
pelajaran dengan kebutuhan hidup sehari-hari maka guru telah berupaya agar
pembelajaran memungkinkan bagi guru untuk mengidentifikasi minat siswa terhadap
.
A.
B.
C.
D.

kesehatan
status
penampilan
pekerjaan ()

2
45. Luas daerah segitiga sama kaki 240 cm dan panjang alasnya 20 cm.
Keliling segitiga adalah ...
A. 68
B. 70
C. 72 ()
D. 78
46. DPR bekerja sama dengan presiden untuk menyusun undang-undang.
Menyusun undang-undang merupakan perwujudan dari fungsi . . . .
A. yudikasi
B. anggaran
C. legislasi ()
D. eksekusi
47. 580 - n = 108 + 460
A. 8
B. 12 ()

n. .
C. 18

D. 22

48. Bacalah teks berikut untuk menjawab soal dibawah ini. Bakat menulis Sony sudah
muncul sejak usia 8 tahun. Kemahirannya tersebut bisa jadi merupakan warisan dari
ayahnya. Ayahnya memang terkenal sebagai seorang penulis yang karya-karyanya
sangat digemari orang. Banyak orang mengatakan bahwa ayahnya adalah seorang
penulis yang hebat. Hal ini disebabkan tulisan-tulisan ayahnya dinilai mampu
membahagiakan hati orang-orang yang membacanya. Kini, meskipun masih kelas VI
SD, Sony sudah menerbitkan belasan novel yang dikarangnya sendiri. Selain sudah
menerbitkan buku, Sony juga selalu menjadi juara dalam setiap lomba yang
diikutinya. Pernah ia menjadi juara dalam lomba menulis cerita anak tingkat kota
madya dan lomba menulis puisi anak tingkat provinsi Jawa Barat. Jika dihitung,
jumlah piagam dan piala yang berderet di rumahnya sudah mencapai lebih dari tiga
puluh buah. Semua prestasi Sony tersebut tentu saja diawali oleh perjuangan dan
ketekunan. Dorongan semangat dari ayahnya memang sangat berpengaruh. Namun,
prestasi Sony takkan dapat diraih tanpa adanya kemauan yang kuat dalam dirinya
untuk tetap menulis. Kelak, ia berharap bisa menjadi seorang penulis terkenal seperti
ayahnya. Ia pun berharap tulisan-tulisannya bisa bermanfaat bagi semua orang,
khususnya anak-anak. Mengapa karya-karya ayah Sony digemari banyak orang?
A. Karena ayah Sony baik hati dan senang menolong orang.
B. Karena karya-karya ayah Sony berisi cerita petualangan.
C. Karena tulisan ayah Sony dinilai dapat membahagiakan hati. ()
D. Karena ayah Sony terkenal sebagai seorang penulis yang hebat.
49. Bentuk persen dari 2/5 adalah .........
A. 50%
B. 10%
C. 40% ()
D. 20
50. Mencari penguatan terhadap keputusan inovasi yang dilakukan termasuk .
A.
B.
C.
D.

Bujukan (persuasion)
Konfirmasi (confirmation) ()
Implementasi (implementation)
Pengembalian keputusan (decision making)

51. Lambang Pecahan desimal dari 3/8 adalah ...


A. 6,25
B. 0,375 ()
C. 37,5
D. 0,0625
52. Setiap guru wajib membuat Rencana Pelaksanaan Pembelajaran.
Pernyataan ini merupakan amanah dari Permendiknas ..
A. Nomor 22 tahun 2006 tentang Standar Isi
B. Nomor 23 tahun 2006 tentang SKL
C. Nomor 41 tahun 2007 tentang Standar Proses ()
D. Nomor 20 tahun 2007 tentang Standar Penilaian Pendidikan
53. Barang dan Jasa yang dikonsumsi dapat digolongkan menjadi 3.
Contoh yang benar dibawah ini adalah barang yang habis satu kali pakai adalah ...
A. makanan, minuman ()
B. minuman
C. makanan
D. pakaian, sabun mandi
54. Aspek positif perilaku konsumtif dibawah ini adalah
A. mengurangi kesempatan menabung
B. menambah lapangan kerja ()
C. hidup boros
D. berprilaku konsumtif
Bacalah teks berikut untuk menjawab soal dibawah ini. Bakat menulis Sony sudah
muncul sejak usia 8 tahun. Kemahirannya tersebut bisa jadi merupakan warisan dari
ayahnya. Ayahnya memang terkenal sebagai seorang penulis yang karya-karyanya
sangat digemari orang. Banyak orang mengatakan bahwa ayahnya adalah seorang
penulis yang hebat. Hal ini disebabkan tulisan-tulisan ayahnya dinilai mampu
membahagiakan hati orang-orang yang membacanya. Kini, meskipun masih kelas VI
SD, Sony sudah menerbitkan belasan novel yang dikarangnya sendiri. Selain sudah
menerbitkan buku, Sony juga selalu menjadi juara dalam setiap lomba yang
diikutinya. Pernah ia menjadi juara dalam lomba menulis cerita anak tingkat kota
madya dan lomba menulis puisi anak tingkat provinsi Jawa Barat. Jika dihitung,
jumlah piagam dan piala yang berderet di rumahnya sudah mencapai lebih dari tiga
puluh buah. Semua prestasi Sony tersebut tentu saja diawali oleh perjuangan dan
ketekunan. Dorongan semangat dari ayahnya memang sangat berpengaruh. Namun,
prestasi Sony takkan dapat diraih tanpa adanya kemauan yang kuat dalam dirinya
untuk tetap menulis. Kelak, ia berharap bisa menjadi seorang penulis terkenal seperti
ayahnya. Ia pun berharap tulisan-tulisannya bisa bermanfaat bagi semua orang,
khususnya anak-anak. Kesimpulan isi bacaan tersebut adalah
A. Setiap orang memiliki bakat terpendam yang hebat.
B. Prestasi dapat diraih melalui perjuangan, ketekunan, dan kemauan kuat. ()
C. Dorongan orang tua sangat penting agar anak menjadi pintar.
D. Kebahagiaan bisa dicapai dengan tekun menulis cerita.
56. Pernyataan di bawah ini merupakan karakteristik perkembangan peserta didik
SD/MI ditinjau dari aspek emosional, kecuali ....
A. kesulitan memulai sesuatu, tetapi jika berhasil akan bertahan sampai akhir
B. menampakkan marah apabila mengalami kesulitan di sekolah

55.

C. mulai muncul perasaan simpati kepada orang yang lebih dewasa ()


D. mulai muncul perasaan simpati kepada orang yang lebih dewasa
57. Hasil dari ( 16 x 3 ) + (20 : 5 ) adalah ....
A. - 52 ()
B. 44
C. 44

D. 52

58. Lembaga negara yang bertugas memeriksa pengelolaan keuangan negara adalah .
...
A. Dewan Pertimbangan Agung
B. Badan Audit Nasional
C. Badan Pemeriksa Keuangan ()
D. Mahkamah Agung
59. Ujung jarum yang selalu menunjukkan arah utara biasanya diberi warna . . . .
A. merah atau hitam ()
B. hijau atau merah C.
merah atau putih D. hitam
atau hijau
60. Hewan berkaki sepuluh = . . . .
A. aves
B. reptilia
C. mamalia
D. dekapoda ()
61. Binatang bersel satu = . . . .
A. aves
B. reptilian

C. mamalia

D. amuba ()

62. Mengingat acara tesebut sangat penting, kami harap Bapak/Ibu datang tepat
pada waktunya. Isi penggalan surat undangan tersebut tepat untuk acara ...
A. adat
B. syukuran
C. rapat ()
D. khitanan

63. Pada mata pelajaran PKn, salah satu minat siswa usia sekolah dasar yang dapat
teridentifikasi dalam proses pembelajaran adalah minat terhadap .
A. masalah sosial ()
B. bidang olahraga
C. tubuh manusia
D. kesehatan manusia
64. Ali, Amir dan Adi mempunyai tali yang masing - masing panjangnya adalah
90 dm, 6,5 m dan 750 cm. Panjang tali ketiga anak tersebut adalah . . . .
A. 21 m
B. 22 m
C . 23 m ()
D. 24 m
65. Batu apung digunakan untuk keperluan ....
A. pengasah kayu supaya halus ()
B. mengasah pisau agar tajam
C. bahan pengeras jalan
D. dinding dan lantai rumah
66. Dalam proses pembelajaran tentang kompetensi dasar: mengidentifikasi
kebutuhan tubuh agar tumbuh sehat dan kuat (makanan, air, pakaian, udara,
lingkungan sehat), guru tampak dapat mengidentifikasi siswa yang selalu bertanya
dan mampu membuat laporan tentang apa saja yang diminatinya dengan sangat
baik. Berdasarkan uraian di atas, dapat dikatakan bahwa guru dapat
mengidentifikasi minat siswa terhadap
A. seks
B. penampilan C. kesehatan ()
D. pekerjaan
67. Tukang nasi goreng adalah salah satu contoh distribusi
A. langsung ()
B. tidak langsung
C. semi langsung
D. semi tidak langsung
68. Proses komunikasi yang dilakukan oleh dua orang atau lebih dimana penerima
pesan dan pengirim pesan mempunyai latar belakang berbeda, baik dari latar
belakang sosial budaya, agama, pendidikan, karakteristik lain disebut . . . .
A. Heterofil ()

B. Homofil

C. Holomofil

D. Homogen

69. Penggunaan ungkapan keras kepala pada kalimat di bawah ini yang tepat adalah
...
A. Anak yang datang itu memang keras kepalanya
B. Dasar anak keras kepala, sulit diatur. ()
C. Si jagoan itu merasa keras kepala.
D. Anak yang baik itu memang keras kepala

70. Himpunan semesta yang tepat dari P = {3, 5, 7, 9, 11, 13} adalah ... .
A. Himpunan bilangan ganjil kurang dari 15
B. Himpunan bilangan Prima kurang dari 15
C. Himpunan bilangan kelipatan 2 kurang dari 15
D. Himpunan bilangan tidak habis dibagi 2 ()
71. Lebah mendapatkan madu, dan bunga terjadi penyerbukan. Kerjasama seperti
ini disebut . . . .
A. simbiosis mutualisme () B.
simbiosis komensalisme C.
simbiosis parasitisme
D. simbiosis sarkasme
72. Bagaimana bunyi butir pertama dasar negara dalam Piagam Jakarta sebelum
diubah?
A. Persatuan Indonesia ()
B. Ketuhanan Yang Maha Esa.
C. Ketuhanan dengan kewajiban menjalankan syariat Islam bagi pemeluk-pemeluknya.
D. Kerakyatan yang dipimpin oleh hikmat kebijaksanaan dalam permusyawaratan
perwakilan.
73. Minat siswa yang dapat teridentifikasi melalui mata pelajaran IPS adalah minat
terhadap lambang status. Salah satunya adalah .
A. nama-nama keluarga
B. penampilan diri
C. ikatan kekerabatan
D. jenis pekerjaan orang tua ()
74. Jumlah bilangan prima dari bilangan antara 25 s/d 50 adalah
A. 71 B. 72 C. 156
D. 187 ()
75. Berikut ini kesulitan yang dimungkinkan muncul pada siswa SD disebabkan
adanya kerusakan-kerusakan pada organ wicara, kecuali .
A. suara B. kosakata () C. artikulasi
D. kelancaran
76. Salah satu nilai juang yang dapat diteladani dari para perumus Pancasila adalah
musyawarah. Apa yang dimaksud dengan musyawarah?
A. Menghargai pendapat yang berbeda atau bahkan bertentangan dengan pendapat
sendiri.
B. Pembahasan bersama dengan maksud mencapai keputusan untuk menyelesaikan
masalah. ()
C. Meninggalkan pendapat sendiri demi mengharapkan pujian dari orang yang berbeda
pendapat.
D. Berjuang sekuat tenaga untuk memenangkan pendapat yang diyakini kebenarannya.

77. Beberapa orang dari buruh Pak Basir bertugas mengandangkan sapi. Makna kata
berecetak miring pada, kalimat tersebut adalah ...
A. memasukkan ()
B. membuat
C. menggiring
D. menyimpan
78. Alkisah, ada sebuah kerajaan bernama Kerajaan Linggar dipimpin oleh seorang
ratu, bernama Ratu
Sima. Ratu Sima mempunyai tiga orang putra, semuanya laki-laki. Putra pertama dan
keduanya lahir
kembar, bernama Pangeran Seta dan Pangeran Seto. Keduanya putra kesayangan
Ratu Sima. Sementara putra ketiganya lahir cacat. Matanya buta. Ia bernama
Pangeran Rasi. Sejak kecil, Pangeran Rasi dibesarkan oleh Mbok Tumi, dayang
kerajaan yang ditugaskan mengasuh Pangeran Rasi. Mereka hidup bahagia, hingga
Pangeran Rasi menjadi pemuda yang baik dan santun budi pekertinya. Kemarau
terus melanda Kerajaan Linggar. Seluruh rakyat sangat berharap turunnya hujan.
Kerajaan Linggar kekeringan. Ada yang mengatakan, Kerajaan Linggar terkena
hukuman karena keserakahan Pangeran Seta dan Pangeran Seto yang menebang
pohon di hutan Randualas, yaitu hutan yang tumbuh di samping Kerajaan Linggar.
Seluruh rakyat tak ada yang berani melaporkan keserakahan kedua pangeran
kembar itu. Semua orang takut karena keduanya putra kesayangan ratu. (Dikutip
dari Cermat Berbahasa, Penerbit Tiga Serangkai) Berdasarkan kutipan dongeng
tersebut, perbedaan watak Pangeran Seta dan Pangeran Rasi adalah ......
A. Pangeran Seta bersifat bijaksana, sedangkan Pangeran Rasi rendah hati. B.
Pangeran Seta bersifat serakah, sedangkan Pangeran Rasi baik hati. () C.
Pangeran Seta bersifat kasar, sedangkan Pangeran Rasi santun.
D. Pangeran Seta bersifat dermawan, sedangkan Pangeran Rasi serakah.
79. Sebuah gedung yang tingginya 30 meter tampak pada layar televisi tingginya 10
cm, maka skalanya adalah ....
A. 1: 200
B. 1: 300 ()
C. 1: 400
D. 1: 500
80. KPK dan FPB dari 32,48 dan 54 adalah ...
A. 864 dan 2 ()
B. 918 dan 2
C. 864 dan 6
D. 918 dan 6

\81. Panjang sebuah kawat tembaga 56 cm dan panjang sebatang kawat besi 104 cm.
Jika kedua kawat itu harus dipotong sama panjang dengan panjang maksimum, dari
kedua kawat itu akan diperoleh
A. 8 batang kawat tembaga dan 8 kawat besi
B. 7 batang kawat tembaga dan 13 kawat besi ()
C. 13 batang kawat tembaga dan 13 kawat besi
D. 14 batang kawat tembaga dan 26 kawat besi
82. Ayam, merak dan burung = . . . .
A. aves () B.
reptilia C.
mamalia
D. dekapoda
83. Batu bara dapat digunakan sebagai bahan pembuat
A. aspal dan lem
B. solar dan pelumas C.
nilon dan kokas () D. oil
dan solar
84. Hasil kerja panitia sembilan adalah
A. sembilan emas jakarta B.
soekarno ketua PPKI C. piagam
jakarta ()
D. Indonesia menang dari penjajah
85. Berikut ini karakteristik anak berkesulitan belajar Matematika yang sering
dialami siswa usia sekolah dasar, kecuali .
A. adanya gangguan hubungan keruangan
B. kesulitan dalam bahasa dan membaca
C. gangguan mengenal dan memahami simbol
D. performance IQ lebih tinggi dari skor Verbal IQ ()
86. Para perumus Pancasila dalam bermusyawarah sangat menghargai perbedaan
pendapat. Mereka menjunjung tinggi . . . .
A. egoism
B. kesukuan
C. toleransi ()
D. kerja keras
87. Dalam proses pembelajaran PKn, guru kelas I SD mengajarkan tentang
kompetensi dasar: Melaksanakan aturan yang berlaku di masyarakat, maka
kompetensi prasyarat yang harus dimiliki siswa adalah .
A. mengenal pentingnya tata tertib di mayarakat
B. melaksanakan hidup rukun di masyarakat
C. mengenal lingkungan rumah dan sekolah
D. mengikuti tata tertib di rumah dan sekolah ()

88. Tahap pada saat individu/kelompok mulai membentuk sikap menyenangi


atau tidak menyenangi terhadap inovasi dikategorikan .................................
A. Tahap pengetahuan (kowledge)
B. Tahap bujukan (persuasion) ()
C. Tahap pengembalian keputusan (decision making)
D. Tahap implementasi (implementation)
89. Mengapa butir pertama dalam Piagam Jakarta diubah?
A. Demi menuruti keinginan Ketua BPUPKI.
B. Demi memenuhi keinginan Mohammad Hatta.
C. Demi menjaga hubungan baik dengan pemerintah Jepang.
D. Demi menjaga persatuan dan kesatuan bangsa yang baru saja dirintis. ()
90.

Adi : Apa saja kegiatan tutup tahun ini kawan


kawan?

Didi : Biasa Di, perpisahan dengan kakak kelas VI.


Yus : Ya, tapi kita dapat usulkan yang lain.
Ani : Hai ini tidak dapat dibicarakan sekilas, kalau mau terima usulku adakan rapat.
Percakapan tersebut menunjukkan bahwa Ani anak yang cermat dan ....
A.
B.
C.
D.

semuanya
tangguh
tanggungjawab ()
laksana

91. Bacalah teks berikut untuk menjawab soal dibawah ini. Bakat menulis Sony
sudah muncul sejak usia 8 tahun. Kemahirannya tersebut bisa jadi merupakan
warisan dari ayahnya. Ayahnya memang terkenal sebagai seorang penulis yang karyakaryanya sangat digemari orang. Banyak orang mengatakan
bahwa ayahnya adalah seorang penulis yang hebat. Hal ini disebabkan tulisan-tulisan
ayahnya dinilai mampu membahagiakan hati orang-orang yang membacanya. Kini,
meskipun masih kelas VI SD, Sony sudah menerbitkan belasan novel yang dikarangnya
sendiri. Selain sudah menerbitkan buku, Sony juga selalu menjadi juara dalam setiap
lomba yang diikutinya. Pernah ia menjadi juara dalam lomba
menulis cerita anak tingkat kota madya dan lomba menulis puisi anak tingkat
provinsi Jawa Barat. Jika dihitung, jumlah piagam dan piala yang berderet di
rumahnya sudah mencapai lebih dari tiga puluh buah. Semua prestasi Sony
tersebut tentu saja diawali oleh perjuangan dan ketekunan. Dorongan semangat
dari ayahnya memang sangat berpengaruh. Namun, prestasi Sony takkan dapat
diraih tanpa adanya kemauan yang kuat dalam dirinya untuk tetap menulis. Kelak,
ia berharap bisa menjadi
seorang penulis terkenal seperti ayahnya. Ia pun berharap tulisan-tulisannya bisa
bermanfaat bagi semua orang, khususnya anak-anak. Di tingkat mana Sony berhasil
menjadi juara lomba menulis puisi anak?
A. provinsi () B.
kecamatan C.
kabupaten D.
Kelurahan

92. Salah satu sifat air yang ditunjukkan gambar dibawah ini adalah

A.
B.
C.
D.

permukaan yang tenang selalu mendatar


bentuknya sesuai dengan wadanya
dapat meresap melalui celah-celah kecil ()
memberikan tekanan di semua arah

93. Hai, Lekaslah ke mari. Tanda baca yang paling tepat digunakan pada akhir
kalimat tersebut adalah ...
A.
B.
C.
D.

tanya
titik
seru ()
titik koma

94. Untuk membelajarkan kompetensi dasar:Perkalian bilangan, kemampuan awal/


prasyarat yang harus dimiliki siswa adalah kemampuan .
A. membagi
B. menjumlah ()
C. mengurang
D. menghitung
95. Bacalah teks berikut untuk menjawab soal dibawah ini. Bakat menulis Sony
sudah muncul sejak usia 8 tahun. Kemahirannya tersebut bisa jadi merupakan
warisan dari ayahnya. Ayahnya memang terkenal
sebagai seorang penulis yang karya-karyanya sangat digemari orang. Banyak orang
mengatakan bahwa ayahnya adalah seorang penulis yang hebat. Hal ini disebabkan
tulisan-tulisan ayahnya dinilai mampu membahagiakan hati orang-orang yang
membacanya. Kini, meskipun masih kelas VI SD, Sony sudah menerbitkan belasan
novel yang dikarangnya sendiri. Selain sudah menerbitkan buku, Sony juga selalu
menjadi juara dalam setiap lomba yang diikutinya. Pernah ia menjadi juara dalam
lombamenulis cerita anak tingkat kota madya dan lomba menulis puisi anak tingkat
provinsi Jawa Barat. Jika dihitung, jumlah piagam dan piala yang berderet di
rumahnya sudah mencapai lebih dari tiga puluh buah. Semua prestasi Sony tersebut
tentu saja diawali oleh perjuangan dan ketekunan. Dorongan semangat dari ayahnya
memang sangat berpengaruh .Namun, prestasi Sony takkan dapat diraih tanpa adanya
kemauan yang kuat dalam dirinya untuk tetap menulis. Kelak, ia berharap bisa
menjadi

seorang penulis terkenal seperti ayahnya. Ia pun berharap tulisan-tulisannya bisa


bermanfaat bagi semua orang, khususnya anak-anak. Bagaimana cara Sony
meraih berbagai prestasi?
A.Sony mewarisi kemahiran menulis dari ayahnya.
B.Sony terus-menerus menulis tanpa mengenalwaktu.
C.Sony meminta dorongan semangat darikeluarganya.
D.Sony berkemauan kuat dan tekun menulis. ()
96. KPK dari bilangan 12 dan 18 adalah ....
A. 12
B. 36 ()
C. 18
D. 216
97. Pemahaman simbol merupakan salah satu kesulitan belajar yang sering
muncul pada siswa. Hal tersebut dapat teridentifikasi melalui pengerjaan soal .
A. 4 + 3 = ..
B. 9 - 6 = ..
C. 8 - = . ()
D. 5 + 4 = .
98. 152 + 52 = n. n adalah . . . .
A.
B.
C.
D.

225
250 ()
325
400

99. Nyamuk, belalang dan lebah = . . . .


A. aves
B. reptilia
C. mamalia
D. serangga ()
100. Air PAM digunakan untuk.
A. keperluan rumah tangga ()
B. pengairan lahan
pertanian
C. pengolahan hasil
hutan
D. pemeliharaan ikan

88. Tahap pada saat individu/kelompok mulai membentuk sikap menyenangi


atau tidak menyenangi terhadap inovasi dikategorikan .................................
A. Tahap pengetahuan (kowledge)
B. Tahap bujukan (persuasion) ()
C. Tahap pengembalian keputusan (decision making)
D. Tahap implementasi (implementation)
89. Mengapa butir pertama dalam Piagam Jakarta diubah?
A. Demi menuruti keinginan Ketua BPUPKI.
B. Demi memenuhi keinginan Mohammad Hatta.
C. Demi menjaga hubungan baik dengan pemerintah Jepang.
D. Demi menjaga persatuan dan kesatuan bangsa yang baru saja dirintis. ()
90.

Adi : Apa saja kegiatan tutup tahun ini kawan


kawan?

Didi : Biasa Di, perpisahan dengan kakak kelas VI.


Yus : Ya, tapi kita dapat usulkan yang lain.
Ani : Hai ini tidak dapat dibicarakan sekilas, kalau mau terima usulku adakan rapat.
Percakapan tersebut menunjukkan bahwa Ani anak yang cermat dan ....
A.
B.
C.
D.

semuanya
tangguh
tanggungjawab ()
laksana

91. Bacalah teks berikut untuk menjawab soal dibawah ini. Bakat menulis Sony
sudah muncul sejak usia 8 tahun. Kemahirannya tersebut bisa jadi merupakan
warisan dari ayahnya. Ayahnya memang terkenal sebagai seorang penulis yang karyakaryanya sangat digemari orang. Banyak orang mengatakan
bahwa ayahnya adalah seorang penulis yang hebat. Hal ini disebabkan tulisan-tulisan
ayahnya dinilai mampu membahagiakan hati orang-orang yang membacanya. Kini,
meskipun masih kelas VI SD, Sony sudah menerbitkan belasan novel yang dikarangnya
sendiri. Selain sudah menerbitkan buku, Sony juga selalu menjadi juara dalam setiap
lomba yang diikutinya. Pernah ia menjadi juara dalam lomba
menulis cerita anak tingkat kota madya dan lomba menulis puisi anak tingkat
provinsi Jawa Barat. Jika dihitung, jumlah piagam dan piala yang berderet di
rumahnya sudah mencapai lebih dari tiga puluh buah. Semua prestasi Sony
tersebut tentu saja diawali oleh perjuangan dan ketekunan. Dorongan semangat
dari ayahnya memang sangat berpengaruh. Namun, prestasi Sony takkan dapat
diraih tanpa adanya kemauan yang kuat dalam dirinya untuk tetap menulis. Kelak,
ia berharap bisa menjadi
seorang penulis terkenal seperti ayahnya. Ia pun berharap tulisan-tulisannya bisa
bermanfaat bagi semua orang, khususnya anak-anak. Di tingkat mana Sony berhasil
menjadi juara lomba menulis puisi anak?
A. provinsi () B.
kecamatan C.
kabupaten D.
Kelurahan

92. Salah satu sifat air yang ditunjukkan gambar dibawah ini adalah

A.
B.
C.
D.

permukaan yang tenang selalu mendatar


bentuknya sesuai dengan wadanya
dapat meresap melalui celah-celah kecil ()
memberikan tekanan di semua arah

93. Hai, Lekaslah ke mari. Tanda baca yang paling tepat digunakan pada akhir
kalimat tersebut adalah ...
A.
B.
C.
D.

tanya
titik
seru ()
titik koma

94. Untuk membelajarkan kompetensi dasar:Perkalian bilangan, kemampuan awal/


prasyarat yang harus dimiliki siswa adalah kemampuan .
A. membagi
B. menjumlah ()
C. mengurang
D. menghitung
95. Bacalah teks berikut untuk menjawab soal dibawah ini. Bakat menulis Sony
sudah muncul sejak usia 8 tahun. Kemahirannya tersebut bisa jadi merupakan
warisan dari ayahnya. Ayahnya memang terkenal
sebagai seorang penulis yang karya-karyanya sangat digemari orang. Banyak orang
mengatakan bahwa ayahnya adalah seorang penulis yang hebat. Hal ini disebabkan
tulisan-tulisan ayahnya dinilai mampu membahagiakan hati orang-orang yang
membacanya. Kini, meskipun masih kelas VI SD, Sony sudah menerbitkan belasan
novel yang dikarangnya sendiri. Selain sudah menerbitkan buku, Sony juga selalu
menjadi juara dalam setiap lomba yang diikutinya. Pernah ia menjadi juara dalam
lombamenulis cerita anak tingkat kota madya dan lomba menulis puisi anak tingkat
provinsi Jawa Barat. Jika dihitung, jumlah piagam dan piala yang berderet di
rumahnya sudah mencapai lebih dari tiga puluh buah. Semua prestasi Sony tersebut
tentu saja diawali oleh perjuangan dan ketekunan. Dorongan semangat dari ayahnya
memang sangat berpengaruh .Namun, prestasi Sony takkan dapat diraih tanpa adanya
kemauan yang kuat dalam dirinya untuk tetap menulis. Kelak, ia berharap bisa
menjadi

seorang penulis terkenal seperti ayahnya. Ia pun berharap tulisan-tulisannya bisa


bermanfaat bagi semua orang, khususnya anak-anak. Bagaimana cara Sony
meraih berbagai prestasi?
A. Sony mewarisi kemahiran menulis dari ayahnya.
B. Sony terus-menerus menulis tanpa mengenalwaktu. C.
Sony meminta dorongan semangat darikeluarganya. D.
Sony berkemauan kuat dan tekun menulis. ()
96. KPK dari bilangan 12 dan 18 adalah ....
A. 12
B. 36 ()
C. 18
D. 216
97. Pemahaman simbol merupakan salah satu kesulitan belajar yang sering muncul
pada siswa. Hal tersebut dapat teridentifikasi melalui pengerjaan soal .
A. 4 + 3 = ..
B. 9 - 6 = ..
C. 8 - = . ()
D. 5 + 4 = .
98. 152 + 52 = n. n adalah . . . .
A.
B.
C.
D.

225
250 ()
325
400

99. Nyamuk, belalang dan lebah = . . . .


A. aves
B. reptilia
C. mamalia
D. serangga ()
100. Air PAM digunakan untuk.
A. keperluan rumah tangga ()
B. pengairan lahan
pertanian
C. pengolahan hasil
hutan
D. pemeliharaan ikan

BERANDA SOAL UKG 2015


AGUS ZAINAL M, S.Pd (SDN GUDANG 2)
1. KD 1.1 Analisis Perkembangan Bahasa Anak Usia SD
Berikut ini adalah Karakteristik Anak Usia SD dari segi Mental, yaitu...
a. Anak sudah memiliki gerakan yang bebas dan aman. Hal ini berguna untuk melakukan berbagai
gerakan motorik kasar (jasmani) seperti memanjat, berlari dan menaiki tangga.
b. Anak dapat menunjukkan kreativitasnya dalam membentuk sesatu karya tertentu

c. Anak mulai tidak suka terikat dengan orang dewasa


d. Anak menunjukkan tenggang rasa dan penghargaan terhadap teman
Uraian:
Segi Mental
- Anak sudah mulai memahami beberapa konsep abstrak seperti menghitung tanpa menggunakan
benda
- Anak sudah dapat menghubungkan suatu objek atau kejadian dengan konsep tertentu yang bersifat
abstrak. Misalnya
tentang luas dan volume
- Anak dapat menunjukkan kreativitasnya dalam membentuk sesatu karya tertentu
- Anak dapat menciptakan sesiatu bentuk/benda dengan menggunakan alat
- Anak dapat membuat gambar-gambar dengan menggunakan sudut perspektif sederhana
- Anak dapat menampilkan sifat ingin tahu
- Anak dapat merumuskan dan menunjukkan pengertian terhadap sesuatu
- Anak sudah dapat mengikuti peraturan yang berlaku umum
- Anak dapat menyelesaikan tugas yang diberikan guru, baik sendiri maupun kerja sama
- Anak dapat menunjukkan aktivitasnya dalam berbagai kegiatan sekolah maupun di lingkungannya
- Anak dapat memperlihatkan insiatif dan alternative untuk memecahkan masalah-masalah tertentu
2. KD 1.1 Analisis Perkembangan Bahasa Anak Usia SD
Berikut ini adalah Karakteristik Anak Usia SD dari segi Psikomotorik, yaitu...
a. Anak sudah dapat memakai pakaian dengan rapi

b. Anak dapat menciptakan sesiatu bentuk/benda dengan menggunakan alat


c. Anak sudah mulai memahami beberapa konsep abstrak seperti menghitung tanpa menggunakan
benda
d. Anak menunjukkan kepedulian terhadap orang lain
Uraian:
Segi Psikomotorik
- Anak sudah memiliki gerakan yang bebas dan aman. Hal ini berguna untuk melakukan berbagai
gerakan motorik kasar
(jasmani) seperti memanjat, berlari dan menaiki tangga.
- Memiliki kemampuan dalam melakukan koordinasi dan keseimbangan badan. Misalnya ketika berjalan
atau berlari
dengan berbagai pola
- Anak sudah dapat memperkirakan kegiatan/gerakan yang berbahaya dan tidak berbahaya
- Anak sudah dapat memakai pakaian dengan rapi
- Anak sudah bias menunjukkan kebersihan dalam berpakaian, badan dan alat-alat yang dibawa
3 KD 1.1 Analisis Perkembangan Bahasa Anak Usia SD
Berikut ini adalah Karakteristik Anak Usia SD dari segi Emosionalitas, yaitu...
a. Anak menunjukkan keceriaan dalam berbagai aktivitas bersama kelompok teman sebayanya

b. Anak dapat memperlihatkan insiatif dan alternative untuk memecahkan masalah-masalah tertentu
c. Anak sudah mulai memahami beberapa konsep abstrak seperti menghitung tanpa menggunakan
benda
d. Anak dapat menampilkan sifat ingin tahu
Uraian:
Segi Sosial Emosionalitas
- Anak mulai tidak suka terikat dengan orang dewasa
- Anak dapat menunjukkan penghargaan terhadap guru atau orang dewasa lainnya
- Anak dapat menunjukkan sikap empati terhadap suatu kondisi
- Anak menunjukkan keceriaan dalam berbagai aktivitas bersama kelompok teman sebayanya
- Anak dapat menunjukkan sikap marah dalam kondisi yang wajar
- Anak menunjukkan kepedulian terhadap orang lain
- Anak menunjukkan tenggang rasa dan penghargaan terhadap teman

- Anak menunjukkan rasa solidaritas terhadap teman sekelompoknya


- Anak telah memiliki kemauan untuk menceritakan sesuatu kepada teman-temannya
4 KD 1.1 Analisis Perkembangan Bahasa Anak Usia SD
Memiliki kemampuan dalam melakukan koordinasi dan keseimbangan badan. Misalnya ketika berjalan
atau berlari dengan berbagai pola adalah karakteristik perkembangan bahasa anak usia SD ditinjau dari
segi ...
a. Mental
b. Psikomotorik

c. Sosial
d. Emosional
Uraian: no 2
5 KD 1.1 Analisis Perkembangan Bahasa Anak Usia SD
Anak dapat menampilkan sifat ingin tahu adalah karakteristik perkembangan bahasa anak usia SD
ditinjau dari segi ...
a. Mental

b. Psikomotorik
c. Sosial
d. Emosional
Uraian:
No. 1
6 KD 1.1 Analisis Perkembangan Bahasa Anak Usia SD
Anak dapat menunjukkan sikap marah dalam kondisi yang wajar adalah karakteristik perkembangan
bahasa anak usia SD
ditinjau dari segi ...
a. Mental
b. Psikomotorik
c. Kognitif
d. Emosional

Uraian:
No. 3
7 KD 1.1 Analisis Perkembangan Bahasa Anak Usia SD
Faktor kendala yang mempengaruhi keterampilan berbahasa anak adalah sebagai berikut, kecuali...
a. Jenis Kelamin
b. Keluarga
c. Keinginan dan Dorongan Komunikasi
d. Kebiasaan

Uraian:
Faktor dan Kendala dalam Mempelajari Keterampilan Berbahasa :
1. Kesehatan.
2. Kecerdasan
3. Jenis kelamin
4. Keluarga
5. Keinginan dan dorongan untuk berkomunikasi serta hubungan dengan teman sebaya.
6. Kepribadian
8 KD 1.1.2 Aspek Membaca di kelas rendah
Tahapan proses belajar membaca bagi siswa sekolah dasar kelas awal biasanya menggunakan metode
membaca
a. Ejaan per ejaan

b. Skimming
c. Permulaan
d. Cepat
Uraian:
Metode membaca permulaan :
1) Metode Eja/alfabet
2) Metode Bunyi
3) Metode Suku Kata
4) Metode Kata
5) Metode Global
6) Metode SAS

9 KD 1.1.2 Aspek Membaca di kelas rendah


Dalam pembelajaran membaca permulaan, ada beberapa metode yang dapat digunakan, kecuali ...
a. metode kupas rangkai suku kata
b. metode kata lembaga

c. metode SAS
d. metode eja
Uraian:
No. 8
10 KD 1.1.2 Aspek Membaca di kelas rendah
Berikut diberikan studi kasus :
Mula-mula diberikan kalimat secara keseluruhan. Kalimat itu diuraikan atas kata-kata yang
mendukungnya. Dari kata-kata itu kita ceraikan atas suku-suku katanya dan akhirnya atas hurufhurufnya. Kemudian huruf-huruf itu kita sintetiskan kembali menjadi suku kata, suku kata menjadi kata
dan kata menjadi kalimat.
Berdasarkan studi kasus , metode membaca permulaan yang tepat digunakan adalah ...
a. Metode Alfabet
b. Metode Suku Kata
c. Metode SAS

d. Metode Cerita
Uraian:
11 KD 1.1.2 Aspek Membaca di kelas rendah
Kelebihan membaca permulaan adalah sebagai berikut ...
a. Mempunyai nilai strategis bagi pengembangan kepribadian dan kemampuan siswa.

b. Meningkatkan nilai siswa dalam pelajaran Bahasa Indonesia


c. Meningkatkan kemandirian siswa dalam membaca
d. Mempermudah menghafal kata-kata
Uraian:
Kelebihan membaca permulaan :
- Dapat memperoleh informasi secara tepat dan lengkap.
- Mempunyai nilai strategis bagi pengembangan kepribadian dan kemampuan siswa.
- Membaca permulaan juga dapat mengembangkan nilai-nilai moral, kemampuan bernalar dan
kreatifitas seseorang.
Kekurangan membaca permulaan :
- Kemampuan membaca pada umumnya lebih rendah.
- Menghambat kemampuan untuk memahami kalimat atau cerita.
- Mengalami kesulitan menjawab mengenai isi cerita karena kesibukan siswa mengeja dan
menyuarakan huruf-huruf.
- Kecepata membaca dan pemahaman siswa sangat rendah.
12 KD 1.1.2 Aspek Membaca di kelas rendah
Metode yang merupakan penyempurnaan metode alphabet dengan mengajarkan bunyi-bunyi bahasa
sebagai pengganti huruf-huruf berdasarkan ucapan hurufnya adalah metode membaca permulaan ..
a. Metode Suku Kata
b. Metode Kata-kata
c. Metode Cerita
d. Metode Suara

Uraian:
Teknik Membaca Permulaan :
1) Metode Alfabet
Metode ini sering disebut metode harfiah, Metode letter by letter method atau ABC method. Mulamula dikenalkan abjad
dari A sampai Z. Setelah hafal beberapa huruf barulah huruf-huruf tersebut dirangkai menjadi suku kata.
Suku kata-suku
kata tersebut setelah dikenalkan barulah dirangkai menjadi kata. Dengan modal, beberapa kata
itu,terciptalah kalimat.
2) Metode Suara
Metode suara juga disebut :Phonic Method. Metode ini merupakan penyempurnaan metode alphabet.
Pada metode ini
bukan abjadnya yang di ajarkan, melainkan bunyi-bunyi bahasa sebagai pengganti huruf-huruf
tersebut.Ucapan huruf-huruf
tidak berdasarkan atas bunyi abjadnya melainkan ucapan hurufnya.

3) Metode Suku Kata


Metode ini biasa juga disebut :Syllabic Method. Dalam metode ini suku kata merupakan kunci pokok
dalam membuat
kata. Metode suku kata sebenarnya suatu metode pengajaran membaca permulaan yang di dasarkan
atas kata-kata
dianalisis menjadi suku kata-suku kata.Suku kata0suku kata tersebut di intesiskan kembali menjadi katakata.
4) Metode Kata-kata
Pelaksanaan metode ini selalu di awali dengan kata-kata tertulis : da da, gi gi, ku ku dan lain-lain.
Setelah kata-kata
itu dikenalkan, kata-kata tersebut dianalisis atas suku katanya. Selanjitnya suku kata itu di analisis lagi
atas huruf-hurufnya.
Barulah huruf-huruf itu digabungkan kembali dalam bentuk kata semula.
5) Metode Cerita
Pelaksanaan metode cerita dalam mengajarkan membaca permulaan diawali dengan menghafalkan
cerita atau sebuah
puisi. Cerita atau puisi itu diuraikan atas kalimat-kalimtnya sampai pada kata-katanya. Dalam
mengucapkan kata0kata
metode ini menggunakan kata-kata fonetik.
6) Metode Struktural Analitik Sintetik (SAS)
Adapun contoh pelaksanaan metode SAS sebagai berikut : Mula-mula diberikan kalimat secara
keseluruhan. Kalimat itu
diuraikan atas kata-kata yang mendukungnya. Dari kata-kata itu kita ceraikan atas suku-suku katanya
dan akhirnya atas
huruf-hurufnya. Kemidian huruf-huruf itu kita sintetiskan kembali menjadi suku kata, suku kata menjadi
kata dan kata
menjadi kalimat.
13 KD 1.1.3 Aspek Menulis di kelas tinggi
Pendekatan menekankan keterpaduan empat aspek keterampilan berbahasa (menyimak, berbicara,
membaca, dan menulis) dalam pembelajaran adalah salah satu Pendekatan yang disarankan dalam
pembelajaran menulis yang disebut ...
a. pendekatan komunikatif
b. Pendekatan integrative

c. Pendekatan keterampilan proses


d. Pendekatan tematis
Uraian:
Pendekatan yang disarankan dalam pembelajaran menulis :
1) Pendekatan komunikatif memfokuskan pada keterampilan siswa mengimplementasikan fungsi
bahasa (untuk berkomunikasi) dalam pembelajaran.
2) Pendekatan integratif menekankan keterpaduan empat aspek keterampilan berbahasa
(menyimak,berbicara,membaca, dan menulis) dalam pembelajaran.
3) Pendekatan keterampilan proses memfokuskan keterampilan siswa dalam mengamati,
mengklasifikasi,menginterpretasi, dan mengkomunikasikan.
4) Pendekatan tematis menekankan tema pembelajaran sebagai payung/pemandu dalampembelajaran
14 KD 1.1.3 Aspek Menulis di kelas tinggi
Teknik menulis cerita terdiri atas hal-hal sebagai berikut, kecuali ...
a. menjawab pertanyaan
b. membuat kalimat
c. subtitusi
d. persuasi

Uraian:
Teknik menyusun cerita dapat dilakukan dengan: menjawab pertanyaan, melengkapi kalimat
memperbaiki susunan kalimat,
memperluas kalimat, subtitusi, transfomtasi dan membuat kalimat.
* Persuasi adalah bujukan halus

15 KD 1.1.3 Aspek Menulis di kelas tinggi


Model pembelajaran menulis cerita/cerpen di SD meliputi hal-hal berikut, kecuali ...
a. Menceritakan gambar
b. Melanjutkan cerita
c. Menceritakan pengalaman
d. Mendeskripsikan cerita

Uraian:
Model pembelajaran menulis cerita/cerpen di SD meliputi: menceritakan gambar, melanjutkan ceria
lain, menceitakan mimpi, menceritakan pengalaman, dan menceritakan cita-cita
16 KD 1.2.1 Memilih berbagai Metode Menulis Permulaan
1). Menulis huruf lepas.
2). Merangkaikan huruf lepas menjadi suku kata.
3). Merangkaikan suku kata menjadi kata.
4). Menyusun kata menjadi kalimat. (Djauzak, 1996:4)
Tahapan diatas adalah Teknik Menulis Permulaan dengan metode ...
a. Metode Eja

b. Metode Kata Lembaga


c. Metode SAS
d. Metode Global
Uraian:
Metode dan pembelajaran menulis permulaan
a. Metode Eja
Metode eja di dasarkan pada pendekatan harfiah, artinya belajar membaca dan menulis dimulai dari
huruf-huruf yang
dirangkaikan menjadi suku kata. Oleh karena itu pengajaran dimulai dari pengenalan huruf-huruf.
Demikian halnya dengan pengajaran menulis di mulai dari huruf lepas, dengan langka-langkah sebagai
berikut:
1). Menulis huruf lepas
2). Merangkaikan huruf lepas menjadi suku kata
3). Merangkaikan suku kata menjadi kata
4). Menyusun kata menjadi kalimat (Djauzak, 1996:4)
b. Metode kata lembaga
Metode kata lembaga di mulai mengajar dengan langkah-langkah sebagai berikut:
1). Mengenalkan kata
2). Merangkaikan kata antar suku kata
3). Menguraikan suku kata atas huruf-hurufnya
4). Menggabungkan huruf menjadi kata (Djauzak, 1996:5)

c. Metode Global
Metode global memulai pengajaran membaca dan menulis permulaan dengan membaca kalimat secara
utuh yang ada dibawah gambar. Menguraikan kalimat dengan kata-kata, menguraikan kata-kata menjadi
suku kata (Djauzak, 1996:6).
d. Metode SAS
Menuryut (Supriyadi, 1996: 334-335) pengertian metode SAS adalah suatu pendekatan cerita di sertai
dengan gambar yang didalamnya terkandung unsur analitik sintetik. Metode SAS menurut
(Djuzak,1996:8) adalah suatu pembelajaran menulis permulaan yang didasarkan atas pendekatan cerita
yakni cara memulai mengajar menulis dengan menampil cerita yang diambil dari dialog siswa dan guru
atau siswa dengan siswa. Teknik pelaksanaan pembelajaran metode SAS yakni keterampilan menulis
kartu huruf, kartu suku kata, kartu kata dan kartu kalimat, sementara sebagian siswa mencari huruf,
suku kata dan kata, guru dan sebagian siswa menempel kata-kata yang tersusun sehingga menjadi
kalimat yang berarti (Subana). Proses operasional metode SAS mempunyai langkah-lagkah dengan
urutan sebagai berikut:
a. Struktur yaitu menampilkan keseluruhan.
b. Analitik yatu melakukan proses penguraian.
c. Sintetik yaitu melakukan penggalan pada struktur semula

17 KD 1.2.1 Memilih berbagai Metode Menulis Permulaan


1). Mengenalkan kata
2). Merangkaikan kata antar suku kata
3). Menguraikan suku kata atas huruf-hurufnya
4). Menggabungkan huruf menjadi kata (Djauzak, 1996:5)
Tahapan diatas adalah Teknik Menulis Permulaan dengan metode ...
a. Metode Eja
b. Metode Kata Lembaga

c. Metode SAS
d. Metode Global
Uraian:
No. 16
18 KD 1.2.1 Memilih berbagai Metode Menulis Permulaan
Metode dengan memulai pengajaran membaca dan menulis permulaan dengan membaca kalimat
secara utuh yang ada dibawah gambar. Menguraikan kalimat dengan kata-kata, menguraikan kata-kata
menjadi suku kata disebut dengan metode..
a. Metode Eja
b. Metode Kata Lembaga
c. Metode SAS
d. Metode Global

Uraian:
No. 16
19 KD 1.2.1 Memilih berbagai Metode Menulis Permulaan
Suatu pembelajaran menulis permulaan yang didasarkan atas pendekatan cerita yakni cara memulai
mengajar menulis dengan menampil cerita yang diambil dari dialog siswa dan guru atau siswa dengan
siswa disebut dengan metode
a. Metode Eja
b. Metode Kata Lembaga
c. Metode SAS

d. Metode Global
Uraian:
No. 16
20 KD 1.2.1 Memilih berbagai Metode Menulis Permulaan
menulis kartu huruf, kartu suku kata, kartu kata dan kartu kalimat, sementara sebagian siswa mencari
huruf, suku kata dan kata, guru dan sebagian siswa menempel kata-kata yang tersusun sehingga
menjadi kalimat yang berarti adalah contoh metode ...
a. Metode Eja
b. Metode Kata Lembaga
c. Metode SAS

d. Metode Global
Uraian: No. 16
21 KD 1.2.2 Merancang berbagai kegiatan menulis di kelas tinggi
Berikut adalah kegiatan menulis lanjutan di kelas tinggi, kecuali...
a. menulis tentang berbagai topik
b. menulis pengumuman
c. menulis pantun
d. menulis memo

Uraian:
22 KD 1.2.3 Perencanaan dan Pelaksanaan evaluasi pembelajaran Bahasa dan Sastra Indonesia
Perencanaan Pengajaran meliputi hal-hal berikut, kecuali...
a. tujuan apa yang hendak dicapai
b. memilih bahan ajar
c. proses belajar mengajar
d. alat penilaian
Uraian:
Menurut Akhlan dan Rahman (1997:15), perencanaan pengajaran meliputi:
a. tujuan apa yang hendak dicapai;
b. bahan pengajaran;
c. proses belajar mengajar;dan
d. alat penilaian.

23 KD 1.2.3 Perencanaan dan Pelaksanaan evaluasi pembelajaran Bahasa dan Sastra Indonesia
karakteristik perencanaan pengajaran yang baik hendaknya mengandung prinsip sebagai berikut...
a. Memiliki sikap objektif rasio (tepat dan masuk akal), komprehensif dan sistematis (menyeluruh dan
tersusun rapi).

b. Merupakan suatu wahana atau wadah untuk mengembangkan segala potensi yang ada dan dimiliki
oleh anak didik.
c. Mengendalikan kekuatan sendiri, bukan didasarkan atas kekuatan orang lain.
d. Melakukan studi kasus yang berkesinambungan.
Uraian:
Menurut Akhlan dan Rahman (1997:7), karakteristik perencanaan pengajaran yang baik hendaknya
mengandung prinsip sebagai berikut:
a. Mengembangkan hubungan interaksi yang baik di antara sesama manusia, dalam hal ini siswa dan
guru serta personal terkait.
b. Merupakan suatu wahana atau wadah untuk mengembangkan segala potensi yang ada dan dimiliki
oleh anak didik.
c. Memiliki sikap objektif rasio (tepat dan masuk akal), komprehensif dan sistematis (menyeluruh dan
tersusun rapi).
d. Mengendalikan kekuatan sendiri, bukan didasarkan atas kekuatan orang lain, Didukung oleh fakta
dan data yang menunjang pencapaian tujuan yang telah di dirumuskan.
e. Fleksibel dan dinamis, artinya mudah disesuaikan dengan keadaan serta perkembangan ke arah
yang lebih baik dan maju.
24 KD 1.2.3 Perencanaan dan Pelaksanaan evaluasi pembelajaran Bahasa dan Sastra Indonesia
tahap mengumpulkan informasi tentang keadaan objek evaluasi (siswa) dengan menggunakan teknik
tes atau nontes disebut tahapan ...
a. Tahap Tindak Lanjut
b. Tahap Persiapan
c. Tahap Pelaksanaan

d. Tahap Pengolahan Hasil


Uraian:
Berikut ini penjelasan singkat tentang keempat tahap evaluasi pembelajaran tersebut.
(1) Tahap Persiapan
Menurut Damaianti (2007: 8) tahap ini disebut juga tahap perencanaan dan perumusan kriterium.
Langkahnya meliputi:
(a) perumusan tujuan evaluasi;
(b) penetapan aspek-aspek yang akan dievaluasi;
(c) menetapkan metode dan bentuk evaluasi (tes/nontes);
(d) merencanakan waktu evaluasi;
(e) melakukan uji coba (untuk tes) agar dapat mengukur validitas dan reliabilitasnya.
Untuk evaluasi yang menggunakan tes, hasil dari tahap ini adalah kisi-kisi soal dan seperangkat alat tes:
soal, lembar
jawaban (untuk tes tulis), kunci jawaban, dan pedoman penilaian.
2) Tahap Pelaksanaan
Tahap pelaksanaan atau disebut juga dengan tahap pengukuran dan pengumpulan data adalah tahap
untuk mengumpulkan informasi tentang keadaan objek evaluasi (siswa) dengan menggunakan teknik
tes atau nontes. Bila menggunakan teknik tes, soal yang igunakan sebaiknya sudah teruji validitas dan
reliabilitasnya. Tes yang digunakan dapat berbentuk tes tulis, lisan, atau praktik.
3) Tahap Pengolahan Hasil
Tahap pengolahan hasil adalah tahap pemeriksaan hasil evaluasi dengan memberikan skor. Skor yang
diperoleh siswa selanjutnya diubah menjadi nilai. Pada tes tulis pemeriksaan hasil dilakukan setelah tes
selesai, sedangkan pada tes lisan dan praktik, pemberian nilai dilakukan bersamaan dengan waktu
pelasanaan tes tersebut.
4) Tahap Tindak Lanjut
Tahap tindak lanjut atau disebut juga tahap penafsiran adalah tahap untuk mengambil keputusan
berdasarkan nilai yang dihasilkan pada tahap pengolahan hasil, misalnya:
a. memperbaiki proses belajar mengajar
b. memperbaiki kesulitan belajar siswa
c. memperbaiki alat evaluasi
d. membuat laporan evaluasi (rapor).

25 KD 1.3.1 Merumuskan hakikat (pengertian,tujuan,jenis,manfaat) membaca


membaca yang mengutamakan isi bacaan sebagai ungkapan pikiran, perasaan, dan kehendak penulis.
Bila hanya ingin mengetahui isinya, membaca cerdas bersifat lugas. Akan tetapi, bila maksudnya untuk
memahami dan memilki isi bacaan, maka tergolong kedalam membaca jenis ...
a. Membaca cerdas atau membaca dalam hati

b. Membaca bahasa
c. Membaca teknis
d. Membaca bebas
Uraian: Menurut Soedjono dalam Sue (2004:18-21) ada lima macam membaca, yaitu: membaca
bahasa, membaca cerdas atau membaca dalam hati, membaca teknis, membaca emosional, dan
membaca bebas.
1) Membaca bahasa
Membaca bahasa adalah membaca yang mengutamakan bahasa bacaan. Membaca bahasa
mementingkan segi bahasa bacaan. Hal-hal yang perlu diperhatikan dalam membaca bahasa adalah
kesesuian pikir dengan bahasa, perbendaharaan bahasa yang meliputi kosa kata, struktur kalimat, dan
ejaan.
2) Membaca cerdas atau membaca dalam hati
Membaca cerdas adalah membaca yang mengutamakan isi bacaan sebagai ungkapan pikiran,
perasaan, dan kehendak
penulis. Bila hanya ingin mengetahui isinya, membaca cerdas bersifat lugas. Akan tetapi, bial
maksudnya untuk memahami dan memilki isi bacaan, maka disebut membaca belajar.
3) Membaca teknis
Membaca teknis adalah membaca dengan mengarahkan bacaan secara wajar. Wajar maksudnya
sesuai ucapan, tekanan, dan intonasinya. Pikiran, perasaan, dan kemauan yang tersimpan dalam
bacaan dapat diaktualisasikan dengan baik.
4) Membaca emosional
Membaca emosional adalah membaca sebagai sarana untuk memasuki perasaan, yaitu keindahan isi,
dan keindahan bahasanya.
5) Membaca bebas
Membaca bebas adalah membaca sesuatu atas kehendak sendiri tanpa adanya unsur paksaan dari
luar. Unsur dari luar misalnya guru, orang tua, teman, atau pihak-pihak lain.
26 KD 1.3.1 Merumuskan hakikat (pengertian,tujuan,jenis,manfaat) membaca
membaca sesuatu atas kehendak sendiri tanpa adanya unsur paksaan dari luar. Unsur dari luar
misalnya guru, orang tua,teman, atau pihak-pihak lain, termasuk jenis membaca
a. Membaca cerdas atau membaca dalam hati
b. Membaca bahasa
c. Membaca teknis
d. Membaca bebas

Uraian:
No. 25
27 KD 1.3.1 Merumuskan hakikat (pengertian,tujuan,jenis,manfaat) membaca
1. menggunakan ucapan yang tepat,
2. menggunakan frase yang tepat,
3. menggunakan intonasi suara yang wajar,
4. dalam posisi sikap yang baik,
5. menguasai tanda-tanda baca,
6. membaca dengan terang dan jelas,
keterampilan di atas, harus dipunyai dalam jenis membaca ...
a. Membaca cerdas atau membaca dalam hati
b. Membaca bahasa
c. Membaca teknis
d. Membaca nyaring

Uraian:
JENIS-JENIS MEMBACA DAN KARAKTERISTIKNYA
Ditinjau dari segi terdengar atau tidaknya suara pembaca waktu melakukan kegiatan membaca, maka
proses membaca dapat dibedakan menjadi :
A. Membaca Nyaring
Membaca nyaring adalah kegiatan membaca dengan menyuarakan tulisan yang dibacanya dengan
ucapan dan intonasi yang tepat agar pendengar dan pembaca dapat menangkap informasi yang
disampaikan oleh penulis, baik yang berupa pikiran, perasaan, sikap, ataupun pengalaman penulis.

Ketrampilan yang dituntut dalam membaca nyaring adalah berbagai kemampuan, diantaranya adalah :
1. menggunakan ucapan yang tepat,
2. menggunakan frase yang tepat,
3. menggunakan intonasi suara yang wajar,
4. dalam posisi sikap yang baik,
5. menguasai tanda-tanda baca,
6. membaca dengan terang dan jelas,
7. membaca dengan penuh perasaan, ekspresif,
8. membaca dengan tidak terbata-bata,
9. mengerti serta memahami bahan bacaan yang dibacanya,
10. kecepatan bergantung pada bahan bacaan yang dibacanya,
11. membaca dengan tanpa terus-menerus melihat bahan bacaan,
12. membaca dengan penuh kepercayaan pada diri sendiri.
B. Membaca Dalam Hati
Membaca dalam hati adalah kegiatan membaca yang dilakukan dengan tanpa menyuarakan isi bacaan
yang dibacanya.
Ketrampilan yang dituntut dalam membaca dalam hati antara lain sebagai berikut:
1. membaca tanpa bersuara, tanpa bibir bergerak, tanpa ada desis apapun,
2. membaca tanpa ada gerakan-gerakan kepala,
3. membaca lebih cepat dibandingkan dengan membaca nyaring,
4. tanpa menggunakan jari atau alat lain sebagai penunjuk,
5. mengerti dan memahami bahan bacaan,
6. dituntut kecepatan mata dalam membaca,
7. membaca dengan pemahaman yang baik,
8. dapat menyesuaikan kecepatan dengan tingkat kesukaran yang terdapat dalam bacaan.
28 KD 1.3.1 Merumuskan hakikat (pengertian,tujuan,jenis,manfaat) membaca
Hal berikut dilakukan seseorang ketika membaca survai , kecuali ...
a. memeriksa judul bacaan/buku, kata pengantar, daftar isi dan malihat abstrak(jika ada)
b. memeriksa bagian terahkir dari isi (kesimpulan) jika ada
c. memeriksa indeks dan apendiks(jika ada)
d. membaca biografi pengarang

Uraian: Secara garis besar, membaca dalam hati dapat dibedakan menjadi dua
(I) MEMBACA EKSTENSIF & (II) MEMBACA INTENSIF. Berikut penjelasan secara rinci kedua jenis
membaca tersebut :
I. Membaca Ekstensif
membaca ekstensif adalah membaca secara luas. Objeknya meliputi sebanyak mungkin teks dalam
waktu yang sesingkatsingkatnya.
Membaca ekstensif meliputi :
1. Membaca Survai (Survey Reading)
Membaca survai adalah kegiatan membaca untuk mengetahui secara sekilas terhadap bahan bacaan
Yang akan dibaca lebih mendalam.Kegiatan membaca survai merupakan pendahuluan dalam
membaca ekstensif. Yang dilakukan seseorang ketika membaca survai adalah sebagai berikut :
(a) Memeriksa judul bacaan/buku, kata pengantar, daftar isi dan malihat abstrak(jika ada),
(b) Memeriksa bagian terahkir dari isi (kesimpulan) jika ada,
(c) Memeriksa indeks dan apendiks(jika ada).
2. Membaca Sekilas
Membaca sekilas atau membaca cepat adalah kegiatan membaca dengan mengandalakan kecepatan
gerak mata dalam melihat dan memperhatikan bahan tertulis yang dibacanya dengan tujuan untuk
mendapatkan informasi secara cepat. Metode yang digunakan dalam melatihkan membaca cepat
adalah :
(a) metode kosakata; metode yang berusaha untuk menambah kosakata.
(b) Metode motivasi; metode yang berusaha memotivasi pembaca(pemula) yang mengalami hambatan.
(c) Metode gerak mata; metode yang mengembangkan kecepatan membaca dengan menigkatkan
kecepatan gerak mata.
Hambatan-hambatan yang dapat mengurangi kecepatan mambaca :
(a) vokalisai atau berguman ketika membaca,

(b) membaca dengan menggerakan bibir tetapi tidak bersuara,


(c) kepala bergerak searah tulisan yang dibaca,
(d) subvokalisasi; suara yang biasa ikut membaca di dalam pikiran kita,
(e) jari tangan selalu menunjuk tulisa yang sedang kit abaca,
(f) gerakan mata kembali pada kata-kata sebelumnya.
3. Membaca Dangkal (Superficial Reading)
membaca dangkal pada hakekatnya bertujuan untuk memperoleh pemahaman yang dangkal yang
bersifat luaran, yang tidak mendalam dari suatu bahan bacaan. Membaca jenis ini biasanya dilakukan
seseorang membaca demi kesenangan, membaca bacaan ringan yang mendatangkan kesenangan,
kegembiraan sebagai pengisi waktu senggang.
29 KD 1.3.1 Merumuskan hakikat (pengertian,tujuan,jenis,manfaat) membaca
Membaca jenis ini biasanya dilakukan seseorang membaca demi kesenangan, membaca bacaan ringan
yang mendatangkan kesenangan, kegembiraan sebagai pengisi waktu senggang.
Berdasarkan karakteristik diatas, kegiatan tersebut termasuk ke dalam membaca jenis ...
a. Membaca Survai (Survey Reading)
b. Membaca Sekilas
c. Membaca Dangkal (Superficial Reading)

d. Membaca Nyaring
Uraian:
No. 28
30 KD 1.3.1 Merumuskan hakikat (pengertian,tujuan,jenis,manfaat) membaca
Berikut adalah tujuan umum dalam aktifitas membaca, kecuali ...
a. Membaca untuk memperoleh perincian-perincian atau fakta-fakta (reading for details or facts).
Membaca tersebut bertujuan untuk menemukan atau mengetahui penemuan-penemuan telah
dilakukan oleh sang tokoh, untuk memecahkan masalah-masalah yang dibuat oleh sang tokoh.
b. Membaca untuk mengetahui ukuran atau susunan, organisasi cerita (reading for sequenceor
organization). Membaca tersebut bertujuan untuk mengetahui bagian-bagian cerita dan hubungan
antar bagian-bagian cerita.
c. Membaca untuk menyimpulkan atau membaca inferensi (reading for inference).
d. Membaca untuk memperoleh kekurangan suatu buku (finding mistakes).

Uraian:
Tujuan Membaca Berdasarkan maksud, tujuan atau keintensifan serta cara dalam membaca di
bawah ini, Anderson dalam Tarigan (1979:910) mengemukakan beberapa Tujuan membaca
Antara lain:
a. Membaca untuk memperoleh perincian-perincian atau fakta-fakta (reading for details or facts).
Membaca tersebut bertujuan untuk menemukan atau mengetahui penemuan-penemuan telah
Dilakukan oleh sang tokoh, untuk memecahkan masalah-masalah yang dibuat oleh sang tokoh.
b. Membaca untuk memperoleh ide-ide utama (reading for main ideas). Membaca untuk mengetahui
topik atau masalah dalam bacaan. Untuk menemukan ide pokok bacaan dengan membaca halamn
demi halaman.
c. Membaca untuk mengetahui ukuran atau susunan, organisasi cerita (reading for sequenceor
organization). Membaca tersebut bertujuan untuk mengetahui bagian-bagian cerita dan hubungan
antar bagian-bagian cerita.
d. Membaca untuk menyimpulkan atau membaca inferensi (reading for inference). Pembaca
diharapkan dapat merasakan sesuatu yang dirasakan penulis.
e. Membaca untuk mengelompokkan atau mengklasifikasikan (reading for classify). Membaca jenis ini
bertujuan untuk menemukan hal-hal yang tidak wajar mengenai sesuatu hal (Anderson dalam
Tarigan 1979:10).
f. Membaca untuk menilai atau mengevaluasai (reading to evaluate). Jenis membaca tersebut
bertujuan menemukan suatu keberhasilan berdasarkan ukuran-ukuran tertentu. Membaca jenis ini
memerlukan ketelitian dengan membandingkan dan mengujinya kembali.
g. Membaca untuk memperbandingkan atau mempertentangkan (reading to compare or contrast).
Tujuan membaca tersebut adalah untuk menemukan bagaimana cara, perbedaan atau persamaan
dua hal atau lebih.

31 KD 1.3.3 menemukan pesan pokok utama sebuah berita


Simak penggalan berita berikut . Hebat, Siswa Indonesia Pertahankan Tradisi Emas di Olimpiade Fisika!
Tim Olimpiade Fisika Indonesia REPUBLIKA.CO.ID, JAKARTA--Luar biasa. Lima siswa Indonesia yang
dikirim ke ajang Olimpiade Fisika atau International Physics Olympiad (IPhO) ke-41 di Zagreb, Kroasia,
17-25 Juli, berhasil menyabet empat medali emas dan satu perak.
Pelajar yang menyumbang emas adalah Muhammad Sohibul Maromi (SMAN 1 Pamekasan, Madura),
Christian George Emor (SMA Lokon St. Nikolaus Tomohon, Sulawesi Utara), David Giovanni (SMAK
Penabur Gading Serpong, Banten), dan Kevin Soedyatmiko (SMAN 12, Jakarta). Sedangkan medali
perak berhasil diraih oleh Ahmad Ataka Awwalur Rizqi (SMAN1, Yogyakarta).
Prestasi ini jauh lebih baik dibanding ajang Olimpiade Fisika ke-40 di Merida Yucatan, Meksiko, 2009
yang lalu. Saat itu,delegasi siswa Indonesia merebut satu medali emas, dua medali perak, dan satu
perunggu. Hasil empat medali emas dan satu perak ini, hampir menyamai prestasi terbaik sebelumnya
pada ajang Olimpiade Fisika ke-37 di Singapura. Saat itu siswa Indonesia tidak hanya berhasil
menyabet 4 medali emas, namun juga meraih predikat `Absolute Winner` atas nama Mailoa Jonathan
Pradana (SMAK 1 BPK Penabur Jakarta). Tapi yang terpenting lagi, pelajar Indonesia berhasil
mempertahankan tradisi emas di setiap ajang Olimpiade Fisika. Red: Endro Yuwanto
Peristiwa yang diberitakan adalah ...
a. siswa Indonesia pertahankan tradisi emas di Olimpiade Matematika
b. siswa Indonesia pertahankan tradisi emas di Olimpiade Fisika
c. siswa Indonesia menyumbang emas di Olimpiade Matematika

d. siswa Indonesia menyumbang perak di Olimpiade Fisika


Uraian:
32 KD 1.3.3 menemukan pesan pokok utama sebuah berita
Simak penggalan berita berikut . Hebat, Siswa Indonesia Pertahankan Tradisi Emas di Olimpiade Fisika!
Tim Olimpiade Fisika Indonesia REPUBLIKA.CO.ID, JAKARTA--Luar biasa. Lima siswa Indonesia yang
dikirim ke ajang Olimpiade Fisika atau International Physics Olympiad (IPhO) ke-41 di Zagreb, Kroasia,
17-25 Juli, berhasil menyabet empat medali emas dan satu perak.
Pelajar yang menyumbang emas adalah Muhammad Sohibul Maromi (SMAN 1 Pamekasan, Madura),
Christian George Emor (SMA Lokon St. Nikolaus Tomohon, Sulawesi Utara), David Giovanni (SMAK
Penabur Gading Serpong, Banten), dan Kevin Soedyatmiko (SMAN 12, Jakarta). Sedangkan medali
perak berhasil diraih oleh Ahmad Ataka Awwalur Rizqi (SMAN 1, Yogyakarta). Prestasi ini jauh lebih baik
dibanding ajang Olimpiade Fisika ke-40 di Merida Yucatan, Meksiko, 2009 yang lalu. Saat itu,
delegasi siswa Indonesia merebut satu medali emas, dua medali perak, dan satu perunggu.
Hasil empat medali emas dan satu perak ini, hampir menyamai prestasi terbaik sebelumnya pada ajang
Olimpiade Fisika ke-37 di Singapura. Saat itu siswa Indonesia tidak hanya berhasil menyabet 4 medali
emas, namun juga meraih predikat `Absolute Winner` atas nama Mailoa Jonathan Pradana (SMAK 1
BPK Penabur Jakarta). Tapi yang terpenting lagi, pelajar Indonesia berhasil mempertahankan tradisi
emas di setiap ajang Olimpiade Fisika. Red: Endro Yuwanto
Lima siswa Indonesia berhasil mendapatkan ...
a. 4 emas , 1 perak

b. 1 emas, 4 perak
c. 3 emas, 2 perak
d. 2 perak , 3 emas
Uraian:
33 KD 1.3.4 menemukan pesan pokok dalam wacana naratif seperti cerita rakyat, puisi
Beranda Soal UKG Agus Zainal M (SDN Gudang 2)
Surat dari ibu
Jika bayang telah pudar
Dan elang laut pulang ke sarang
Angin bertiup ke benua
Tiang tiang akan kering sendiri
Dan nahkoda sudah tau pedoman
Boleh engkau datang padaku
Makna lambang dari nahkoda sudah tahu pedoman adalah
a. Sudah mencari pedoman hidup
b. Sudah menemukan arah dan tujuan

c. Sudah berilmu dan berpengalaman


d. Sudah mempunyai pasangan hidup
Uraian:

34 KD 1.3.5 membandingkan berbagai jenis wacana bahasa indonesia (deskripsi narasi)


Pendekatan untuk mendapat tanggapan emosional pembaca ataupun kesan pembaca adalah contoh
Pendekatan Deskripsi
jenis ...
a. Pendekatan Ekspositoris.
b. Pendekatan Impresionistik

c. Pendekatan menurut sikap pengarang


d. Pendekatan Realistik
Uraian: PENDEKATAN DESKRIPSI
Pendekatan dalam pendeskripsian dapat dibedakan menjadi beberapa kategori pendekatan yaitu:
1. Pendekatan Ekspositoris
Dalam pendekatan ini kita berusaha agar deskripsi yang kita buat dapat memberi keterangan sesuai
dengan keadan yang sebenarnya, sehingga pembaca dapat seolah-olah ikut melihat atau merasakan
objek yang kita deskripsikan. Karangan jenis ini berisi daftar, detail sesuatu secara lengkap sehingga
pembaca dan penalarannya dapat memperoleh kesan keseluruhan tentang sesuatu.

2. Pendekatan Impresionistik
Tujuan deskripsi impresionistik ialah untuk mendapatkan tanggapan emosional pembaca ataupun kesan
pembaca. Corak deskripsi ini diantaranya juga ditentukan oleh macam kesan apa yang diinginkan
penulisnya.
3. Pendekatan menurut Sikap Pengarang
Pendekatan ini sangat bergantung pada tujuan yang ingin dicapai, sifat objek, serta pembaca
deskripsinya. Dalam menguraikan sebuah gagasan penulis mungkin mengharapkan agar pembaca
merasa tidak puas terhadapa suatu tindakan atau keadaan, atau penulis menginginkan agar pembaca
juga harus merasakan bahwa persoalan yang dihadapi merupakan masalah yang gawat. Penulis juga
dapat membayangkan bahwa akan terjadi sesuatu yang tidak diinginkan,sehingga pembaca dari semula
sudah disiapkan perasaan yang kurang enak, seram, takut, dan sebagainya (Alkhaidah,1997).
35 kd 1.3.5 membandingkan berbagai jenis wacana bahasa indonesia (deskripsi narasi)
Prinsip-Prinsip karangan Narasi sebagai berikut, kecuali ...
a. Alur
b. Penokohan
c. Amanat
d. Sudut Pandang
Uraian:
Prinsip-prinsip dasar narasi sebagai tumpuan berfikir adalah sebagai berikut:
1. Alur (plot) biasa disebut dengan jalan cerita
2. Penokohan (rangkaian perbuatan/aksi)
3. Latar (setting) yaitu tempat dan atau waktu terjadinya perbuatan tokoh atau peristiwa yang
dialami tokoh.
4. Sudut Pandang (Point of View)
a. Narrator Serba Tahu (Omniscient point of view)
Yaitu narrator bertindak sebagai pencipta segalanya.
b. Narrator bertindak objektif (Objektive point of view)
Pengarang tak member komentar apa pun dan pembaca bebas menafsirkan apa yang
diceritakan pengarang.
c. Narrator ikut aktif ( Narator acting)
Narrator terlibat dalam cerita.
d. Narrator sebagai peninjau Pelakunya adalah orang ketiga yang biasa disebut dia
36 KD 1.3.5 membandingkan berbagai jenis wacana bahasa indonesia (deskripsi narasi)
1. Menentukan tema atau amanat apa yang akan disampaikan.
2. Menetapkan sasaran pembaca.
3. Merancang peristiwa-peristiwa utama yang akan ditampilkan dalam bentuk skema
perkembangan,dan akhir cerita.
5. Memerinci peristiwa-peristiwa utama ke dalam detail-detail peristiwa sebagai pendukung cerita.
6. Menyusun tokoh dan perwatakan,serta latar dan sudut pandang.

Karakteristik diatas adalah pengembangan dari paragraf...


a. Narasi

b. Deskprisi
c. Ekspositoris
d. Sudut Pandang Persuasif
Uraian:
Pengembangan karangan narasi dapat dilakukan dengan langkah-langkah berikut:
(1) menentukan tema atau amanat apa yang akan disampaikan,
(2) menetapkan sasaran pembaca,
(3) merancang peristiwa-peristiwa utama yang akan ditampilkan dalam bentuk skema alur,
(4) membagi peristiwa utama ke dalam ke dalam bagian awal, perkembangan, dan akhir cerita,
(5) memerinci peristiwa-peristiwa utama ke dalam detail-detail peristiwa sebagai pendukung cerita, dan
(6) menyusun tokoh dan perwatakan, latar, dan sudut pandang
37 KD 1.3.5 membandingkan berbagai jenis wacana bahasa indonesia (deskripsi narasi)
Karangan narasi yang berusaha untuk memberikan suatu maksud tertentu, menyampaikan suatu
amanat terselubung kepada para pembaca atau pendengar sehingga tampak seolah-olah melihat
disebut ...
a. Narasi Informatif
b. Narasi Ekspositoris
c. Narasi Sugestif
d. Narasi Artistik

Uraian:
Jenis-Jenis Narasi :
Narasi Informatif
Narasi informatif adalah narasi yang memiliki sasaran penyampaian informasi secara tepat tentang
suatu peristiwa dengan tujuan memperluas pengetahuan orang tentang kisah seseorang.
Narasi Ekspositorik
Narasi ekspositorik adalah narasi yang memiliki sasaran penyampaian informasi secara tepat tentang
suatu peristiwa dengan tujuan memperluas pengetahuan orang tentang kisah seseorang. Dalam narasi
ekspositorik, penulis menceritakan suatu peristiwa berdasarkan data yang sebenarnya. Pelaku yang
ditonjolkan biasanya, satu orang. Pelaku diceritakan mulai dari kecil sampai saat ini atau sampai terakhir
dalam kehidupannya. Karangan narasi ini diwarnai oleh eksposisi, maka ketentuan eksposisi juga
berlaku pada penulisan narasi ekspositprik. Ketentuan ini berkaitan dengan penggunaan bahasa yang
logis, berdasarkan fakta yang ada, tidak memasukan unsursugestif atau bersifat objektif.
Narasi Artistik
Narasi artistik adalah narasi yang berusaha untuk memberikan suatu maksud tertentu,
menyampaikan suatu amanat terselubung kepada para pembaca atau pendengar sehingga tampak
seolah-olah melihat. Ketentuan ini berkaitan dengan penggunaan bahasa yang logis, berdasarkan fakta
yang ada, tidak memasukan unsur sugestif atau bersifat objektif.
Narasi Sugestif
Narasi sugestif adalah narasi yang berusaha untuk memberikan suatu maksud tertentu,
menyampaikan suatu amanat terselubung kepada para pembaca atau pendengar sehingga tampak
seolah-olah melihat.
38 KD 1.3.5 membandingkan berbagai jenis wacana bahasa indonesia (deskripsi narasi)
Ciri karangan Narasi yang benar, kecuali ...
a. Menonjolkan unsur perbuatan atau tindakan, Membuat pembaca atau pendengar merasakan sendiri
atau mengalami sendiri.
b. Ada konfiks, menjawab pertanyaan "apa yang terjadi?"
c. Dirangkai dalam urutan waktu, menggambarkan dengan jelas suatu peristiwa
d. Berisi ajakan, dirangkai dalam urutan waktu

Uraian:
Ciri-Ciri Karangan Narasi Menurut Gorys Keraf (2000:136) :
Menonjolkan unsur perbuatan atau tindakan.
Dirangkai dalam urutan waktu.
Berusaha menjawab pertanyaan "apa yang terjadi?"
Ada konfiks

39 kd 1.3.5 membandingkan berbagai jenis wacana bahasa indonesia (deskripsi narasi)

Langkah menyusun paragraf deskripsi yang benar adalah ...


(1).Tentukan objek atau tema yang akan dideskripsikan.
(2).Mengumpulkan data dengan mengamati objek yang akan dideskripsikan.
(3).Menyusun data tersebut ke dalam urutan yang baik (menyusun kerangka karangan).
(4).Menguraikan kerangka karangan menjadi dekripsi yang sesuai dengan tema yang ditentukan.
(5).Tentukan tujuan
a. 1-5-2-3-4

b. 1-5-4-3-2
c. 1-5-3-2-4
d. 1-3-4-2-5
Uraian: Langkah menyusun deskripsi:
1.Tentukan objek atau tema yang akan dideskripsikan.
2.Tentukan tujuan.
3.Mengumpulkan data dengan mengamati objek yang akan dideskripsikan.
4.Menyusun data tersebut ke dalam urutan yang baik (menyusun kerangka karangan).
5.Menguraikan kerangka karangan menjadi dekripsi yang sesuai dengan tema yang ditentukan
40 Kd 3.1.1 Merancang aktivitas pembelajaran berdasarkan prinsip dan teori pembelajaran matematika
Dalam tahap ini penyajian yang dilakukan melalui tindakan anak secara langsung terlihat dalam
memanipulasi (mengotak atik)objek. Proses tersebut menurut Bruner dikategorikan ke dalam model ...
a. Tahap Enaktif

b. Tahap Ikonik
c. Tahap Simbolik
d. Tahap Implikatif
Uraian:
1. Model Tahap Enaktif
Dalam tahap ini penyajian yang dilakukan melalui tindakan anak secara langsung terlihat dalam
memanipulasi (mengotak atik)objek. Contoh : Budi mempunyai 2 pensil, kemudian ibunya
memberikannya lagi 3 pinsil. Berapa banyak pensil Budi sekarang ?
2. Model Tahap Ikonik
Dalam tahap ini kegiatan penyajian dilakukan berdasarkan pada pikiran internal dimana pengetahuan
disajikan melalui serangkaian gambar-gambar atau grafik yang dilakukan anak, berhubungan dengan
mental yang merupakan gambaran dari objek-objek yang dimanipulasinya. Contoh : + =
3. Model Tahap Simbolis
Dalam tahap ini bahasa adalah pola dasar simbolik, anak memanipulasi Simbol-simbol atau lambanglambang objek tertentu. Contoh : 2 pensil + 3 pensil = pensil
41 Kd 3.1.1 Merancang aktivitas pembelajaran berdasarkan prinsip dan teori pembelajaran matematika
Dalam tahap ini kegiatan penyajian dilakukan berdasarkan pada pikiran internal dimana pengetahuan
disajikan melalui serangkaian gambar-gambar atau grafik yang dilakukan anak
Proses tersebut menurut Bruner dikategorikan ke dalam model ...
a. Tahap Enaktif
b. Tahap Ikonik

c. Tahap Simbolik
d. Tahap Implikatif
Uraian:
No. 40
42 Kd 3.1.1 Merancang aktivitas pembelajaran berdasarkan prinsip dan teori pembelajaran matematika
Objek belajar matematika dibagi kedalam Objek Langsung dan Objek Tak Langsung.
Hal tersebut adalah teori belajar Matematika menurut ...
a. Robert M. Gagne

b. Jerome S. Burner
c. Thorndike
d. Skinner
Uraian:
Teori yang diperkenalkan Robert M.Gagne pada tahun 1960-an pembelajaran harus dikondisikan untuk
memunculkan respons yang diharapkan.Menurut Gagne (dalam Ismail 1998), belajar matematika terdiri
dari objek langsung dan objek tak langsung.
43 Kd 3.1.1 Merancang aktivitas pembelajaran berdasarkan prinsip dan teori pembelajaran matematika
Perhatikan contoh berikut. Budi mempunyai 2 pinsil, kemudian ibunya memberikannya lagi
pinsil.Berapa banyak pinsil Budi sekarang ?
Hal tersebut dikemukakan Bruner dalam Proses Pembelajaran Matematika dalam tahap ...

a. Simbolik
b. Ikonik
c. Implikatif
d. Enaktif

Uraian:
No. 40
44 Kd 3.1.1 Merancang aktivitas pembelajaran berdasarkan prinsip dan teori pembelajaran
matematika
Perhatikan gambar disamping Ilustrasi di samping dikemukakan Bruner dalam Proses Pembelajaran
Matematika dalam tahap
a. Simbolik
b. Ikonik

c. Implikatif
d. Enaktif
Uraian:
No. 40
45 Kd 3.1.1 Merancang aktivitas pembelajaran berdasarkan prinsip dan teori pembelajaran matematika
Perhatikan contoh berikut. Contoh : 2 pinsil + 3 pinsil = pinsil
Contoh tersebut dikemukakan Bruner dalam Proses Pembelajaran Matematika dalam tahap ...
a. Simbolik

b. Ikonik
c. Implikatif
d. Enaktif
Uraian:
No. 40
46 Kd 3.1.1 Merancang aktivitas pembelajaran berdasarkan prinsip dan teori pembelajaran matematika
Perhatikan contoh berikut. penjumlahan bilangan positif dan negatif siswa mencoba sendiri dengan
menggunakan garis bilangan. Contoh tersebut dikemukakan Bruner dalam Teorema Pembelajaran
Matematika ...
a. Penyusunan

b. Notasi
c. Pengkontrasan dan Keanekaragaman
d. Pengaitan
Uraian:
Berdasarkan hasil pengamatannya, Brunner merumuskan 5 teorema dalam pembelajaran matematika,
yaitu :
1) Teorema Penyusunan
Menerangkan bahwa cara yang terbaik memulai belajar suatu konsep matematika, dalil, defenisi, dan
semacamnya adalah dengan cara menyusun penyajiannya. Misalnya dalam mempelajari penjumlahan
bilangan positif dan negatif siswa mencoba sendiri dengan menggunakan garis bilangan.
2) Teorema Notasi
Menerangkan bahwa dalam pengajaran suatu konsep, penggunaan notasi-notasi matematika harus
diberikan secara bertahap, dari yang sederhana ke yang lebih kompleks.
3) Teorema Pengkontrasan dan Keanekaragaman
Menerangkan bahwa pengontrasan dan keanekaragaman sangat penting dalam melakukan
pengubahan konsep matematika dari yang konkrit ke yang lebih abstrak. Dalam hal ini diperlukan
banyak contoh. Contoh yang diberikan harus sesuai dengan rumusan yang diberikan. Misalnya
menjelaskan persegi panjang, disertai juga kemungkinan jajaran genjang dan segi empat lainnya selain
persegi panjnag. Dengan demikian siswa dapat membedakan apakah segi empat yang diberikan
padanya termasuk persegi panjang atau tidak.
4) Teorema Pengaitan
Menerangkan bahwa dalam matematika terdapat hubungan yang berkaitan antara satu konsep dengan
konsep yang lain. Di mana materi yang satu merupakan prasyarat yang harus diketahui untuk
mempelajari materi yang lain.
47 Kd 3.1.1 Merancang aktivitas pembelajaran berdasarkan prinsip dan teori pembelajaran matematika
Perhatikan contoh berikut.
Guru menjelaskan persegi panjang, disertai juga kemungkinan jajaran genjang dan segi empat lainnya
selain persegi panjnag. Dengan demikian siswa dapat membedakan apakah segi empat yang diberikan
padanya termasuk persegi panjang atau tidak.

Contoh tersebut dikemukakan Bruner dalam Teorema Pembelajaran Matematika ...

a. Penyusunan
b. Notasi
c. Pengkontrasan dan Keanekaragaman

d. Pengaitan
Uraian:
No. 46
48 Kd 3.1.1 Merancang aktivitas pembelajaran berdasarkan prinsip dan teori pembelajaran matematika
Tahap pembelajaran Matematika menurut Van Halle adalah sebagai berikut, kecuali ..
a. Tahap Pengenalan
b. Tahap Pengurutan
c. Tahap Analisis
d. Tahap Penyimpulan

Uraian:
Van Hiele menyatakan bahwa terdapat 5 tahap belajar siswa dalam belajar geometri, yaitu :
a. Tahap Pengenalan
Pada tahap ini siswa mulai belajar mengenal suatu bangun geometri secara keseluruhan namun belum
mampu mengetahui adanya sifat-sifat dari bangun geometri yang dilihatnya.
b. Tahap Analisis
Pada tahap ini siswa sudah mulai mengenal sifat-sifat yang dimiliki bangun geometri yang diamatinya.
c.
Tahap Pengurutan
Pada tahap ini siswa sudah mengenal dan memahami sifat-sifat suatu bangun geometri serta sudah
dapat mengurutkan bangun-bangun geometri yang satu sama yang lainnya saling berhubungan.
d. Tahap Deduksi
Pada tahap ini siswa telah mampu menarik kesimpulan secara deduktif, yaitu menarik kesimpulan yang
bersifat umum dan menuju ke hal yang bersifat khusus serta dapat mengambil kesimpulan.
e.
Tahap Akurasi
Pada tahap ini siswa mulai menyadari pentingnya ketepatan prinsip-prinsip dasar yang melandasi suatu
pembuktian.Tahap berfikir ini merupakan tahap berfikir yang paling tinggi, rumit, dan kompleks, karena di
luar jangkauan usia anakanak SD sampai tingakat SMP
49 Kd 3.1.1 Merancang aktivitas pembelajaran berdasarkan prinsip dan teori pembelajaran matematika
Matematika untuk tujuan pembelajaran dianalisis sebagai kumpulan fakta yang berdiri sendiri
dan tidak saling berkaitan.
Anak diharuskan menguasai unsur-unsur yang banyak sekali tanpa diperhatikan pengertiannya.
Anak mempelajari unsur-unsur dalam bentuk seperti yang akan digunakan nanti dalam
kesempatan lain.
Anak akan mencapai tujuan ini secara efektif dan efisien dengan melalui pengulangan.
Teori Pembelajaran Matematika tersebut dikemukakan oleh ...
a. Skinner
b. Piaget
c. Van Brownell

d. Thorndike
Uraian:
Brownell mengemukakan tentang Teori Makna (Meaning Theory) sebagai pengganti Teori Latihan
Hafal/Ulangan (Drill Theory). Intisari dari teori Drill adalah :
- Matematika untuk tujuan pembelajaran dianalisis sebagai kumpulan fakta yang berdiri sendiri
dan tidak saling berkaitan.
- Anak diharuskan menguasai unsur-unsur yang banyak sekali tanpa diperhatikan pengertiannya.
- Anak mempelajari unsur-unsur dalam bentuk seperti yang akan digunakan nanti dalam
kesempatan lain.
- Anak akan mencapai tujuan ini secara efektif dan efisien dengan melalui pengulangan
50 Kd 3.1.1 Merancang aktivitas pembelajaran berdasarkan prinsip dan teori pembelajaran matematika
Tahap operasional/operasi konkrit adalah teori Pembelajaran Matematika yang dikemukakan Peaget
pada tahap usia ...
a. dibawah 2 tahun
b. 2-7 Tahun
c. 7-8 Tahun
d. 7-12 Tahun

Uraian:

Piaget membagi skema yang digunakan anak untuk memahami dunianya melalui empat periode utama
yang berkorelasi dengan dan semakin canggih seiring pertambahan usia:
Periode sensorimotor (usia 02 tahun)
Periode praoperasional (usia 27 tahun)
Periode operasional konkrit (usia 712 tahun)
Periode operasional formal (usia 12 tahun sampai dewasa)
51 3.1.3 Memilih media pembelajaran yang tepat untuk pembelajaran operasi bilangan bulat
Untuk operasi bilangan bulat perkalian , media yang paling tepat digunakan adalah ...
a. sapu lidi

b. kerikil
c. daun pakis
d. koin
Uraian:
52 3.1.3 Memilih media pembelajaran yang tepat untuk pembelajaran operasi bilangan bulat
Untuk operasi bilangan bulat negatif, kita bisa menggunakan media ...
a. uang kertas
b. kerikil
c. mistar
d. koin
Uraian:
53 3.1.3 Memilih media pembelajaran yang tepat untuk pembelajaran operasi bilangan bulat
Untuk operasi bilangan bulat penjumlahan, media yang paling tepat digunakan adalah ...
a. uang kertas
b. kerikil
c. daun pakis
d. koin

Uraian:
54 3.1.4 Memilih media pembelajaran yang tepat untuk pembelajaran operasi bilangan pecahan
Untuk operasi bilangan pecahan, kita bisa menggunakan media berikut, kecuali ...
a. kartu bilangan
b. garis bilangan
c. gambar bidang
d. blok pecahan

Uraian:
55 3.1.6. Memilih media pembelajaran yang tepat untuk pembelajaran geometri dan pengukuran
Untuk menghitung luas dan keliling bangun datar, media yang paling tepat digunakan adalah ...
a. Penggaris
b. Kertas Folio Bergaris

c. Neraca
d. Kalkulator
Uraian:

UJI KOMPETENSI GURU PENJASKES 100 SOAL


100 Questions I By Ismuadi
Pilih salah satu jawaban yang tepat dengan cara klik pada lingkaran di sebelah kiri. Jika terjadi salah
pilih, Anda dapat memperbaiki jawaban beberapa kali tanpa mengurangi nilai. Waktu ujian akan
tampil di layar; perhatikan waktu yang tersisa. Jika waktu habis sebelum selesai, pekerjaan Anda
akan ditutup dan dikoreksi secara otomatis. Klik tombol NEXT pada soal terakhir untuk menyudahi
pekerjaan Anda dan sekaligus melihat hasilnya. Silakan berdoa terlebih dahulu. Selamat
mengerjakan, semoga berhasil
A.
B.
C.
D.
2.
A.
B.
C.
D.

menangkap makna secara penuh


mengingat kembali kata-kata
membandingkan informasi yang diterima
merumuskan alternatif pemecahan masalah
Suhu badan normal manusia adalah.
40C
39C
38C
37C

3. Imunisasi ulang diberikan khusus wanita kelas VI SD sebanyak 2 kali berturut-turut dengan
tenggang waktu 4 minggu adalah.
A.
Depteri tetanus
B.
Tetanus texoid
C.
Bacillus calmette quiren
D.
Defteri pertusis tetanus
4. Ketika dalam pembelajaran Bahasa Indonesia, ketika ada siswa yang sangat antusias memerankan
tokoh dongeng atau cerita rakyat yang disukai berarti guru telah menyajikan bahan pelajaran
dengan menarik, dengan demikian guru dapat mengindentifikasi minat siswa terhadap.
A.
penampilan
B.
pekerjaan
C.
sekolah
D.
status
5.
A.
B.
C.
D.
6.
A.
B.
C.
D.

Tangki septic adalah tempat menampung.


Sampah
Limbah
Air buangan dapur
tinja
Lari termasuk cabang olahraga..
Atletik
Senam
Binaraga
beladiri

7. Selain anjing, binatang lain yang dapat menyebabkan rabies adalah..


A.
Kera
B.
Kodok
C.
Nyamuk
D.
Ikan

Kompetensi yang harus dimiliki guru (pendidik) berdasarkan Permendiknas Nomor 16 tahun
8. 2007 adalah kompetensi ..

A.
B.
C.
D.

Kepribadian, pedagogik, professional, dan manajerial.


Kepribadian, pedagogik, professional, dan social.
Kepribadian, pedagogik, professional, dan kewirausahaan.
Pedagogik, professional, manajeria, dan sosial.

9. Kreativitas merupakan salah satu karakteristik perkembangan intelektual siswa SD, yang artinya
kemampuan untuk .
A.
memecahkan masalah-masalah dalam kehidupan sehari-hari yang
sering dilakukan dan menghasilkan kepuasan kepada dirinya
sendiri dan orang lain
B.
penalaran yang menggunakan logika-logika yang dapat diterima
oleh semua orang dan menghasilkan penyelesaian persoalan untuk
mengambil keputusan
C.
berfikir tentang sesuatu dengan suatu cara yang baru dan tidak
biasa serta menghasilkan penyelesaian yang unik terhadap berbagai
persoalan
D.
mengembangkan ide-ide secara cerdas dalam rangka penyelesaian
masalah-masalah yang dihadapi dalam kehidupan masa sekarang
maupun masa yang akan datang
10. Salah satu sifat perubahan dalam inovasi yaitu upaya penyusunan kembali berbagai komponen
yang ada dalam siustem dengan maksud untuk menyesuaikan dengan tuntutan dan kebutuhan,
disebut . . . .
A.
Addition
B.
Restructuring
C.
Alternation
D.
Substitution
11. Setiap guru wajib membuat Rencana Pelaksanaan Pembelajaran. Pernyataan ini merupakan
amanah dari Permendiknas ..
A.
Nomor 22 tahun 2006 tentang Standar Isi
B.
Nomor 23 tahun 2006 tentang SKL
C.
Nomor 41 tahun 2007 tentang Standar Proses
D.
Nomor 20 tahun 2007 tentang Standar Penilaian Pendidikan
12. Ruang Lingkup UKS meliputi pernyataan dibawah ini, kecuali :
A.
Penyelenggaraan pendidikan kesehatan
B.
Penyelenggaraan pelayanan kesehatan
C.
Pembinaan lingkungan kehidupan sekolah sehat
D.
Pembinaan Tim Olah Raga yang solid
13.
A.
B.
C.
D.

Berikut ini adalah karakteristik sikat gigi yang baik, kecuali :


bulu sikatnya keras sehingga mampu menjangjau seluruh lapisan gigi.
bulu sikatnya tidak terlalu keras dan tidak terlalu lunak
permukaan bulu sikat gigi rata, kepala sikat gigi kecil,
tangkai sikat gigi lurus.

14. Zat pembangun adalah zat gizi yang diperlukan tubuh untuk membangun atau untuk melakukan
pertumbuhan. Yang bukan termasuk Zat Pembangun adalah
A.
Protein
B.
Mineral
C.
Air
D.
Karbohidrat
15. Pernyataan di bawah ini merupakan karakteristik perkembangan peserta didik SD/MI ditinjau
dari aspek fisik, kecuali ....
A.
menunjukkan variasi yang besar pada tinggi dan berat badan
B.
memiliki keterampilan fisik untuk memainkan permainan
C.
penambahan-penambahan dalam kemampuan motorik halus
D.
memiliki kemampuan dalam mengangkat beban yang berat

16. Salah satu jenis permainan tradisional yang kita pelajari adalah.
A.
Rounders

B.
C.
D.

Kasti
Kippers
Gobag sodor

17. Sumber makanan yang banyak B mengandung vitamin C adalah


A.
Kangkung, daun singkong, bayam

B.
C.

Jeruk, tomat, jambu


Wortel, pepaya, nanas

D.

Semangka, salak, bayam

18. Dalam proses pembelajaran tentang kompetensi dasar: mengidentifikasi kebutuhan tubuh agar
tumbuh sehat dan kuat (makanan, air, pakaian, udara, lingkungan sehat), guru tampak dapat
mengidentifikasi siswa yang selalu bertanya dan mampu membuat laporan tentang apa saja yang
diminatinya dengan sangat baik. Berdasarkan uraian di atas, dapat dikatakan bahwa guru dapat
mengidentifikasi minat siswa terhadap
A.
seks
B.
penampilan
C.
kesehatan
D.
pekerjaan
19. Dalam proses pembelajaran PKn, guru kelas I SD mengajarkan tentang kompetensi dasar:
Melaksanakan aturan yang berlaku di masyarakat, maka kompetensi prasyarat yang harus
dimiliki siswa adalah .
A.
mengenal pentingnya tata tertib di mayarakat
B.
melaksanakan hidup rukun di masyarakat
C.
mengenal lingkungan rumah dan sekolah
D.
mengikuti tata tertib di rumah dan sekolah
20. Berikut ini karakteristik anak berkesulitan belajar Matematika yang sering dialami siswa usia
sekolah dasar, kecuali .
A.
adanya gangguan hubungan keruangan
B.
kesulitan dalam bahasa dan membaca
C.
gangguan mengenal dan memahami simbol
D.
performance IQ lebih tinggi dari skor Verbal IQ
21. Pemahaman simbol merupakan salah satu kesulitan belajar yang sering muncul pada siswa. Hal
tersebut dapat teridentifikasi melalui pengerjaan soal .
A.
4 + 3 = ..
B.
9 - 6 = ..
C.
8 - = .
D.
5 + 4 = .
22. Pada mata pelajaran PKn, salah satu minat siswa usia sekolah dasar yang dapat teridentifikasi
dalam proses pembelajaran adalah minat terhadap .
A.
masalah sosial
B.
bidang olahraga
C.
tubuh manusia
D.
kesehatan manusia
23.
A.
B.
C.
D.

Kaporit dalam system penjernihan air berfungsi sebagai.


Menghilangkan bau
Menghilangkan warna
Mengharumkan air
Membunuh kuman.

24. Berikut ini kesulitan yang dimungkinkan muncul pada siswa SD disebabkan adanya kerusakankerusakan pada organ wicara, kecuali .

A.
B.
C.
D.

suara
kosakata
artikulasi
kelancaran

25. Berikut ini adalah pernyataan yang benar mengenai Memelihara Kebersihan Pribadi, kecuali :
A
Upaya memelihara kebersihan pribadi peserta didik tidak terlepas dari upaya pendidikan
.
secara keseluruhan dan pendidikan kesehatan pada khususnya, karena menjaga kebersihan
pribadi secara optimal, tidak mungkin dapat terwujud tanpa adanya penanaman sikap
hidup bersih dan contoh teladan dari orangtua dan masyarakat sekitarnya.
B
Pendidikan kebersihan adalah salah satu upaya pendidikan yang diberikan di sekolah dan
.
di lingkungan rumah tinggal.
C
Memelihara kebersihan pribadi perlu dukungan peralatan yang memadai karena tubuh kita
.
senantiasa bergerak dan berkeringat sehingga kuman mudah masuk atau menempel di
tubuh kita.
D
Yang diharapkan dari kebersihan pribadi adalah agar peserta didik mengetahui akan
.
manfaat dan pentingnya kebersihan pribadi an mampu membersihkan bagian-bagian
tubuh, serta mampu menerapkan perawatan kebersihan pribadi dalam upaya peningkatan
kesehatan pribadi.
26. Jika terjadi hands ball di daerah lapangan tengah hukumannya adalah.
A.
Trow in
B.
Kick of
C.
Free kick
D.
Goal kick
27. Pada usia remaja putra, pertumbuhan fisik akan terus bertambah hingga usia.
A.
21 tahun
B.
20 tahun
C.
18 tahun
D.
19 tahun
28. Sejumlah kemampuan yang harus dimiliki peserta didik dalam mata pelajaran tertentu sebagai
rujukan untuk menyusun indikator kompetensi adalah :
A.
Standar Kompetensi
B.
Kompetensi Dasar
C.
Kompetensi Dasar Mata Pelajaran
D.
Standar Kompetensi Lulusan
29. Dengan telah diberlakukannya Undang-Undang No. 32 Tahun 2004, maka berbagai program
pelaksanaan UKS di setiap daerah pada dasarnya diserahkan sepenuhnya kepada
.. untuk menentukan prioritas programnya
A.
Tim Pembina UKS di daerahnya masing-masing
B.
Kepala Sekolah
C.
Dinas Pendidikan Tingkat Kecamatan
D.
Dinas Pendidikan Tingkat Kabupaten
30.
A.
B.
C.
D.

Kick off dilakukan di.


Tengah lapangan
Pinggir lapangan
Depan gawang
Penjuru lapangan

31. Materi Pendidikan Kesehatan di SD yang masuk dalam SAINS pada KBK adalah sebagai
berikut, KECUALI :
A.
Kebersihan dan kesehatan pribadi
B.
Makanan bergizi.
C.
Pendidikan kesehatan reproduksi.
D.
Sistem pernapasan

32.
A.

Induk organisasi renang Indonesia adalah


PRSI

B.
C.
D.

PSII
ISSI
IPSI

33. Mencegah selalu lebih baik dan murah daripada mengobati. Oleh karena itu, penting sekali
mengusahakan agar setiap orang dapat berbuat dan melakukan usaha pencegahan, antara lain
seperti dibawah ini, KECUALI
A
Memelihara dan meningkatkan kebersihan, serta menjauhkan diri dari sumber penyakit
.
sehingga terhindar dari penularan.
B
Memeriksakan kesehatan diri pribadi secara teratur dalam jangka waktu tertentu,sekurang.
kurangnya dua kali dalam setahun.
C
Meningkatkan daya tahan tubuh terhadap serangan penyakit, misalnya dengan jalan
.
pengebalan (vaksinasi) dan selalu makan makanan yang bergizi sesuai dengan kebutuhan.
D
Hanya makan makanan yang memiliki standar kesehatan dari Kementerian Kesehatan
.
sehingga asupan gizi yang masuk ke tubuh kita memenuhi standar minimal.
34.
A.
B.
C.
D.

Sasaran Pembinaan UKS adalah seperti dibawah ini, kecuali :


Peserta didik dan Pembina UKS
RPP dan Silabus di sekolah termasuk metodologi mengajar
Sarana dan prasarana pendidikan kesehatan dan pelayanan kesehatan
Lingkungan

35. Standar nasional pendidikan yang berkaitan dengan perencanaan, pelaksanaan, dan pengawasan
kegiatan pendidikan pada tingkat satuan pendidikan, kabupaten/kota, provinsi, atau nasional agar
tercapai efisien dan efektivitas penyelenggaraan pendidikan disebut .
A.
Standar Proses.
B.
Standar Kompetensi Lulusan.
C.
Standar Sarana dan Prasarana.
D.
Standar Pengelolaan.
36. Agar penyakit tidak mudah menular / menyebar di sekitar kita, maka kita harus
A.
Membiarkan sampah menumpuk
B.
Membuang sampah dimana saja
C.
Selalu menjaga kebersihan lingkungan
D.
Membiarkan saluran air kumuh
37. Apabila terjadi pelanggaran handsball di titik A maka hukumannya.
A.
Kick off
B.
Free kick
C.
Corner kick
D.
Corner kick
38. Dalam proses pembelajaraan Matematika, khususnya ketika guru mengaitkan materi pelajaran
dengan kebutuhan hidup sehari-hari maka guru telah berupaya agar pembelajaran
memungkinkan bagi guru untuk mengidentifikasi minat siswa terhadap .
A.
kesehatan
B.
status
C.
penampilan
D.
pekerjaan
39. Frekuensi pencucian rambut sangat tergantung kepada hal-hal berikut, kecuali :
A.
Keadaan cuaca apakah dalam keadaan lembab atau kering
B.
Tebal atau tipisnya rambut, semakin tebal harus pula semakin sering dicuci.
C
Lingkungan atau tempat berada seseorang, misalnya pada lingkungan yang berdebu orang
.
tersebut harus sering mencuci rambutnya.
D.
Seseorang yang sering memakai minyak rambut harus pula sering mencuci rambutnya.

40. Tujuan Pendidikan Kebersihan Pribadi adalah seperti dibawah ini, kecuali :
A
Meningkatkan pengetahuan peserta didik mengenai masalah kebersihan dan hubungannya

.
B
.
C
.
D
.

dengan kesehatan perseorangan, kesehatan keluarga, dan kesehatan masyarakat.


Mengubah sikap mental peserta didik kearah positif yang akan mendorong mereka agar
secara sadar mencintai kebersihan, berbuat dan berperilaku sesuai dengan prinsip hidup
bersih dan sehat dalam kehidupan sehari-hari.
Meningkatkan keterampilan peserta didik yang akan memungkinkan mereka memiliki
kemampuan untuk hidup bersih, baik untuk dirinya sendiri maupun untuk kepentingan
keluarga dan lingkungannya.
Meningkatkan pengetahuan peserta didik mengenai teknik bermain satu cabang olahraga
sehingga memungkinkan siswa berprestasi secara professional.

41. Zat pengatur adalah zat gizi yang berfungsi mengatur metabolism (proses kerja tubuh). Yang
bukan termasuk kelompok zat pengatur adalah
A.
air
B.
karbohidrat
C.
vitamin
D.
mineral.
42. Perhatikan soal-soal di bawah ini. 75
68 27
13 _____ _ _____ + 58
71
Jawaban soal di atas menunjukkan salah satu kesalahan umum yang sering dilakukan oleh anak
usia SD yang berkesulitan belajar Matematika dalam hal .
A.
proses penghitungan
B.
memahami simbol
C.
nilai tempat
D.
bahasa dan membaca
43. Usaha Kesehatan Sekolah adalah wahana untuk meningkatkan kemampuan hidup sehat dan
derajat kesehatan peserta didik sedini mungkin. UKS merupakan perpaduan dua upaya dasar
yaitu
A.
upaya pendidikan dan upaya kesehatan.
B.
Upaya pendidikan dan pelatihan
C.
Upaya olah raga dan kesehatan
D.
Upaya olah raga dan Pramuka
44. Proses komunikasi yang dilakukan oleh dua orang atau lebih dimana penerima pesan dan
pengirim pesan mempunyai latar belakang berbeda, baik dari latar belakang sosial budaya,
agama, pendidikan, karakteristik lain disebut . . . .
A.
Heterofil
B.
Homofil
C.
Holomofil
D.
Homogen
45. Inovasi merupakan suatu perubahan baru yang secara kualitatif berbeda dengan yang ada
sebelumnya, disengaja, untuk meningkatkan kemampuan dalam mencapai tujuan. Pernyataan
tersebut dikemukakan oleh . . . .
A.
Santosa S. Hamidjaja
B.
Mattew B. Milers
C.
Sumantri Mulyani
D.
Rabbin
46.
A.
B.
C.
D.

Induk organisasi bola basket di Indonesia adalah.


Persani
Perbasi
PSSI
Persetasi

47. Pembelajaran Kontekstual dianggap pembelajaran yang bersifat mendidik, hal itu disebabkan
karena karakteristik pembelajaran tersebut adalah .
A.
Membangkitkan minat siswa.
B.
Merangsang siswa lebih aktif belajar.
C.
Memberi kesempatan siswa berkreasi.
D.
Membiasakan siswa belajar sesuai dengan lingkungan.
48. Pengoreksi sikap dan bentuk tubuh yang salah adalah tujuan dari.
A.
Senam artistic

B.
C.
D.
49.
A.
B.
C.
D.
50.
A.
B.
C.
D.
51.
A.
B.
C.
D.

Senam ritmik
Senam irama
General gymnastic
Handspring termasuk gerakan
Atletik
Renang
Senam
Silat
Berat peluru yang dilombakan untuk putra senior adalah
4 kg
4,26 kg
7 kg
7,26 kg
Alat pengukur suhu tubuh adalah.
Barometer
Thermometer
Stopwatch
Speedometer

52. Puskesmas adalah salah satu bentuk pelayanan kesehatan pemerintah yang dilaksanakan pada
tingkat.
A.
RW
B.
Desa
C.
Kecamatan
D.
Kotamadya
53.
A.
B.
C.
D.

Posisi di depan penjaga gawang adalah


Back
Playmaker
Striker
Keeper

54. Usaha Kesehatan Sekolah dijadikan sebagai usaha meningkatkan kesehatan anak usia sekolah
pada setiap jalur, jenis dan jenjang pendidikan, KECUALI :
A.
TK/RA
B.
Pendidikan Dasar SD/MI
C.
Pendidikan menengah SMP/MTs dan SMA/SMK/MA.
D.
Mahasiswa Perguruan Tinggi
55.
A.
B
.
C.
D.
56.
A.
B.
C.
D.
57.
A.
B.
C.
D.

58.
A.

UKS merupakan salah satu wahana untuk .


Memupuk kerjasama dan toleransi diantara para siswa sedini mungkin
Meningkatkan kemampuan hidup sehat dan derajat kesehatan peserta didik sedini
mungkin.
Meningkatkan tingkat kecerdasan dan keahlian siswa sedini mungkin
Memupuk jiwa kepemimpinan siswa sedini mungkin
Permainan bola basket berasal dari.
Inggris
Belanda
Amerika
Norwegia
Alat yang dipergunakan untuk senam lantai adalah.
Papan
Ubin
Matras
Bale

Ventilasi udara dalam rumah berguna untuk.


Pergantian cahay matahari

B.
C.
D.

Pergantian suasana
Pergantian udara
Ke luar masuknya masuk

59. Perkembangan perilaku moral dan perkembangan konsep moral merupakan fase-fase
perkembangan moral yang harus dicapai seorang anak. Pada fase perkembangan perilaku moral,
seorang anak belajar melalui cara-cara berikut, kecuali .
A.
coba-ralat (trial and error)
B.
pendidikan langsung,
C.
identifikasi
D.
observasi
60. Salah satu kegiatan social yang dilakukan dalam posyandu adalah antaralain
A.
Penyuluhan gizi
B.
Operasi kelahiran
C.
Perawatan penyakit kronis
D.
Pemberian obat
61. Pada anak usia sekolah dasar sering disebut usia berkelompok. Pernyataan tersebut
menunjukkan karakteristik perkembangan anak dalam aspek .
A.
sosial
B.
moral
C.
intelektual
D.
emosional
62. Upaya pertama dan yang paling utama agar seseorang dapat tetap dalam keadaan sehat adalah
dengan ..
A.
Menjaga kesehatan di lingkungan keluarga
B.
Menjaga kesehatan di lingkungan sekolah
C.
Menjaga kebersihan dan kesehatan diri sendiri
D.
Menjaga kesehatan di lingkungan sekitar
63.
A.
B.
C.
D.

Induk organisasi atletik tingkat dunia adalah.


FIFA
IAAF
FIBA
FINA

64.
A.
B.
C.
D.

Zat makanan yang berfungsi sebagai sumber tenaga adalah


Protein, mineral, air
Vitamin, mineral, air
Karbohidrat, lemak, protein
Lemak, vitamin, air

65.
A.
B.
C.
D.

Tujuan pemanasan sebelum melakukan latihan, kecuali


Menghindari tejadinya cedera
Meningkatkan suhu badan
Menyiapkan fisik dan mental
Untuk memenangkan pertandingan

66. Organisasi Tim Pembina UKS pada tingkatan pemerintahan secara berjenjang diatur sebagai
berikut, KECUALI :
A
Tim Pembina UKS Pusat, dibentuk di tingkat Pusat dan ditetapkan oleh Mendiknas,
.
Menkes, Menag, dan Mendagri (SKB 4 Menteri);
B
Tim Pembina UKS tingkat sekolah dibentuk di tingkat sekolah dan ditetapkan oleh Kepala
.
Sekolah
C.
Tim Pembina UKS Provinsi, dibentuk di tingkat Provinsi dan ditetapkan oleh Gubernur;
D
Tim Pembina UKS Kabupaten/Kota, dibentuk di tingkat Kabupaten/Kota dan ditetapkan
.
oleh Bupati/Walikota;

67.
A.

Gerakan renang yang menyerupai hewan adalah


Gaya bebas

B.
C.
D.

Gaya dada
Gaya punggung
Gaya kupu-kupu

68. Berikut ini adalah pernyataan yang benar mengenai Pertumbuhan, kecuali..................
A
Pertumbuhan berkaitan dengan berkembangnya kemampuan dalam struktur dan fungsi
.
tubuh yang lebih kompleks dalam pola yang teratur dan dapat diperkirakan
B
Pertumbuhan berkaitan dengan masalah perubahan dalam besar, jumlah, ukuran atau
.
dimensi tingkat sel
C
Pertumbuhan berkaitan dengan masalah perubahan organ maupun individu, yang biasanya
.
diukur dengan ukuran berat (kilogram), ukuran panjang ( cm, meter), umur tulang dan
keseimbangan
D.
Pertumbuhan mempunyai dampak terhadap aspek fisik.
69. Pelaksanaan pendidikan kesehatan pada jam pelajaran sesuai dengan Garis-garis Besar Program
Pengajaran mata pelajaran sains dan ilmu pengetahuan social adalah pelaksanaan pendidikan
melalui ..
A.
kegiatan kurikuler
B.
kegiatan ekstrakurikuler
C.
Palang Merah Remaja
D.
Muatan Lokal
70. Pengukuran pertumbuhan perlu dilakukan untuk menentukan apakah tumbuh kembang
seseorang berjalan normal atau tidak. Anak yang sehat akan menunjukan pertumbuhan yang
optimal. Manfaat pengukuran pertumbuhan adalah sebagai berikut KECUALI :
A
Sebagai bahan informasi untuk menilai keadaan kekurangan gizi baik yang akut maupun
.
yang kronis.
B
Basis data fisik dan pribadi anak sebagai dasar pengiriman ke cabang olahraga yang
.
relevan
C.
Memonitor keadaan kesehatan, misalnya pada pengobatan penyakit.
D.
Dasar perhitungan dosis obat dan makanan yang perlu diberikan.
71. Untuk membelajarkan kompetensi dasar:Perkalian bilangan, kemampuan awal/ prasyarat yang
harus dimiliki siswa adalah kemampuan .
A.
membagi
B.
menjumlah
C.
mengurang
D.
menghitung
72.
A.
B.
C.
D.
73.
A.
B.
C.
D.

Jarak yang ditempuh dalam lari estafet adalah.


4 x 300 m
4 x 400 m
1.600 m
4 x 500 m
Imunisasi mempunyai sifat.
Preventif
Kuratif
Rehabilitative
Presentatif

74. Pernyataan di bawah ini merupakan karakteristik perkembangan peserta didik SD/MI ditinjau
dari aspek emosional, kecuali ....
A.
kesulitan memulai sesuatu, tetapi jika berhasil akan bertahan
sampai akhir
B.
menampakkan marah apabila mengalami kesulitan di sekolah
C.
mulai muncul perasaan simpati kepada orang yang lebih dewasa
D.
mulai muncul perasaan simpati kepada orang yang lebih dewasa

75.

Tempra adalah salah satu obat.

A.
B.
C.
D.

Diare
Dehidrasi
Turun panas
Cacingan

76. Apabila terjadi pelanggaran di depan goal area oleh pemain bertahan maka hukumannya
adalah.
A.
Goal kick
B.
Penalty
C.
Free kick
Corner
D.
kick
77. Minat siswa yang dapat teridentifikasi melalui mata pelajaran IPS adalah minat terhadap
lambang status. Salah satunya adalah .
A.
nama-nama keluarga
B.
penampilan diri
C.
ikatan kekerabatan
D.
jenis pekerjaan orang tua
78. Tim UKS di Sekolah/Madrasah dan Perguruan Agama memiliki tugas dan wewenang selaku :
A.
Tim Pembina
B.
Tim Pelaksana
C.
Tim Pengawas
D.
Tim Pemantau
79. Tujuan Usaha Kesehatan Sekolah (UKS) adalah seperti tersebut dibawah ini, kecuali :
A
meningkatkan mutu pendidikan dan prestasi belajar peserta didik melalui peningkatan
.
kualitas derajad kesehatan
B
meningkatkan perilaku hidup bersih dan sehat serta derajat kesehatan peserta didik
.
maupun warga belajar
C
menciptakan lingkungan yang sehat, sehingga memungkinkan pertumbuhan dan
.
perkembangan yang harmonis dan optimal dalam rangka pembentukan manusia Indonesia
seutuhnya.
D
menciptakan tim olahraga yang solid yang siap berprestasi baik di tingkat local maupun
.
internasional
80.
A.
B.
C.
D.

Satu regu dalam permainan sepak bola adalah.


10 orang
11 orang
12 orang
13orang

81. Untuk membelajarkan kompetensi dasar: mendeskripsikan bendabenda di sekitar dan fungsi
anggota tubuh dengan kalimat sederhana, maka kemampuan awal/ prasyarat yang harus dimiliki
siswa adalah kemampuan .
A.
membuat karangan deskriptif
B.
melengkapi kalimat belum selesai
C.
menyusun kalimat sederhana
D.
membaca kalimat sederhana
82.
A.
B.
C.
D.
83.
A.
B.
C.
D.

84.

Dalam lari kemenangan ditentukan oleh.


jarak lari
timer
waktu tempuh
starter
Lari estafet disebut juga.
Suttle run
Steaplecease
Sprint
Berantai

Jumlah pelari tiap regu lari estafet adalah

A.
B.
C.
D.
85.

1 orang
2 orang
3 orang
4 orang
1. Ciri-ciri air yang dapat dikonsumsi kita adalah.
A. Terasa manis
B. Tidak berwarna dan tidak berbau
C. Berbau dan berwarna
Pahit kadang manis

A.
B.
C.
D.
86.
A.
B.
C.
D.

Terasa manis
Tidak berwarna dan tidak berbau
Berbau dan berwarna
Pahit kadang manis
Orang bertugas mengukur kecepatan lari disebut.
Starter
Timer
Sprinter
Juri

87. Kuku jari tangan maupun jari kaki harus selalu dipelihara kebersihannya. Ciri-ciri kuku yang
sehat adalah seperti dibawah ini kecuali :
A.
Permukaan agak kasar sebagai pertanda kuku yang sehat
B.
Kuku tumbuh dengan baik
C.
Kuat
D.
Bersih
88. Mencari penguatan terhadap keputusan inovasi yang dilakukan termasuk . . . .
A.
Bujukan (persuasion)
B.
Konfirmasi (confirmation)
C.
Implementasi (implementation)
D.
Pengembalian keputusan (decision making)
89. Dibawah Ini yang tidak termasuk menu makanan empat sehat lima sempurna adalah :
A.
Sepiring nasi sebagai sumber zat tenaga
B.
Sepotong daging atau ikan atau sepotong tempe sebagai sumber zat pembangun
C.
Semangkuk sayur
D.
Segelas es kelapa muda untuk menggantikan ion tubuh yang hilang
90.
A.
B.
C.
D.

Start yang digunakan pelari 100 meter adalah.


Standing start
Flying start
Start melayang
Buch start

91.
A.
B.
C.
D.

Induk organisasi sepak bola dunia adalah.


FIFA
FINA
FIBA
IAAF

92. Tahap pada saat individu/kelompok mulai membentu sikap menyenangi atau tidak menyenangi
terhadap inovasi dikategorikan .................................
A.
Tahap pengetahuan (kowledge)
B.
Tahap bujukan (persuasion)
C.
Tahap pengembalian keputusan (decision making)
D.
Tahap implementasi (implementation)

93.
A.
B.
C.
D.

Gaya gunting adalah gaya dalam.


Lompat jauh
Lompat tinggi
Lompat indah
Lompat jangkit

94. Kemampuan awal/prasyarat yang harus dimiliki siswa sebelum mempelajari kompetensi dasar:
Mendeskripsikan gejala (peristiwa) alam yang terjadi negara tetangga, adalah .
A.
mendeskripsikan gejala/peristiwa alam yang terjadi di Indonesia
B.
mendeskripsikan gejala/peristiwa alam yang terjadi di Eropa
C.
mendeskripsikan gejala/peristiwa alam yang terjadi di Asia
D.
mendeskripsikan gejala/peristiwa alam yang terjadi di Afrika
95. Dalam usaha peningkatan kesehatan, masalah kebiasaan hidup bersih, menyenangi kebersihan
dan keserasian harus ditanamkan sejak dini, yaitu :
A.
Sejak bayi masih berada dalam kandungan
B.
dari kelas satu sekolah dasar, bahkan sejak ditaman kanakkanak (pra sekolah).
C.
Sejak kelas 3 SD
D.
Sejak kelas 5 SD
96. Perut terasa mual, nafsu makan berkurang, tidak bergairah, wajah pucat, lesu, perut buncit, dan
gatal-gatal di sekitar dubur adalh tand-tanda penyakit.
A.
Kurang gizi
B.
TBC
C.
Cacingan
D.
diare
97.
A.
B.
C.
D.

Urutan latihan untuk mencapai kebugaran jasmani adalah.


Latihan inti, latiahan pemanasan, latihan pendinginan
Latihan pemanasan, latihan pendinginan, latihan inti
Latihan pemanasan, latihan inti, latihan pendinginan
Latihan pendinginan, latihan pemanasan, latihan inti

98. Usia remaja yang sudah dapat mengendalikan emosi adalah.


A.
Remaja pemuda
B.
Remaja yunior
C.
Remaja senior
D.
Remaja usia puber
99.
A.
B.
C.
D.

Pelaksanaan Pendidikan Kesehatan diberikan melalui :


Kegiatan kurikuler
Kegiatan ekstrakurikuler
Kegiatan kurikuler dan kegiatan ekstrakurikuler
Melalui Perkemahan Sabtu Malam Minggu

100. Pernyataan di bawah ini yang merupakan karakteristik perkembangan peserta didik SD/MI
ditinjau dari aspek sosial adalah....
A.
mulai menyukai teman sebaya sesama jenis
B.
berperan serta dalam permainan logika
C.
menyukai teman sebaya lawan jenis
D.
dapat bekerja dalam durasi waktu yang lama

Anda mungkin juga menyukai